Download as pdf or txt
Download as pdf or txt
You are on page 1of 40

Sample Booklet

Chapter - 4

Motion in Two Dimensions

position of the body as a whole. Consequently, in some cases the


description of the motion of a body is reduced to the description of
the motion of a point.
Various types of motion of a point differ first of all in the shape of the
paths. If the path is a straight line, the motion of the point is referred
to as rectilinear (motion in a straight line or motion in one
dimension). If the path is a curve, the motion is said to be curvilinear
(motion in two or three dimensions). For instance, the centre of the
wheel rolling on a horizontal road in Fig. 3.2 moves in a straight line,
while point P is in a curvilinear motion.
Point P in the above example moves in a plane, so does
a piece of stone thrown obliquely in air, the swinging
bob of a simple pendulum and a girl in a merry-go-
round. Can you give a few more examples? In this
chapter we shall study the motion of bodies in a two-
dimensional plane.

Rectilinear motion is only a special case - the 4.2. Displacement, Velocity and Acceleration
simplest one indeed. But the bodies do not The quantities like displacement, velocity, and
always move in straight lines. A roller acceleration were introduced in Chapter - 3 for one-
coaster employs an elevated railroad track with tight dimensional motion. We must generalize them to two
turns, steep slopes and sometimes inversions. dimensions emphasizing their vector nature.
Let us consider a simple example. As depicted in
Fig. 4.1, a girl strolling on a ground moves from point
4.1. Motion in a Plane A to point B along a path (Path 1) of length 60 m.
Before going into the specifics of motion in a plane we After some time she finds herself at point A again. She
reproduce two paragraphs from Chapter - 3 to begin goes to point B for the second time, now along a
with. different path (Path 2) of length 40 m. After a while the
girl again walks from point A to point B, covering a
Let us consider an example from everyday life. Make a mark on the
rim of your bicycle and focus on the mark when you ride it. What is 90-m distance along Path 3.
the trajectory of the mark as seen by you? Is it the rim circumference Path 1 60 m
itself? What is the trajectory of the mark relative to a person standing
on the ground? Figure 3.2 shows the trajectory of a point (P) on the Path 2 40 m
rim of a rolling wheel relative to the ground.
A B

Path 3 90 m
P
Fig. 4.1
P In the three laps of motion described above the girl
Fig. 3.2 moves from point A to point B - along three different
To describe the motion of a body, we must know how its various paths covering three different distances.
points move. Many a time we are interested only in the change of the

66
What is common to these three laps of motion of the What is the total distance the girl traveled? It is the sum
girl? The staring point A, from where she starts moving of the lengths of the paths 1 and 2: total distance is
and point B, where she halts. We can say that the initial d  60 m  50 m  110 m.
position of her motion is point A and the final position
Path 1 60 m
is B. Her position changes from point A to point B, in
each lap.
It turns out that this change in position is an extremely A 30 m
useful quantity in describing the motion of bodies. We B
call this quantity displacement. Thus displacement is
change in position of a moving body. 25 m
C Path 2
We list here a few incorrect definitions of displacement
we have been hearing from students over the years. 50 m
*Displacement is the shortest distance between two points.
*Displacement is the shortest distance between the initial and
Fig. 4.2
final positions of a moving body. What is the net change in the position of the girl? How
*Displacement is the straight line connecting the initial and to represent it? It can be represented by an arrow drawn
final positions of a moving body. from her initial position A to the final position C . Is
These statements do not qualify to be definition of a the length of this arrow equal to the sum of the lengths
physical quantity, the clauses in them do not signify of the arrows AB and BC , 30 m  25 m  55 m ?
something essentially meaningful. These clauses
assume specific meanings in certain situations. Students Most unlikely. It follows that displacements cannot be
are required to give these statements a thought and added numerically or algebraically. They belong to
figure out why they cannot be definition of some other class with its own set of rules for
displacement. manipulation.
Let us represent the three displacements, from A to B ,
But as we understand, science is not about coining   
words and definitions. It is about experiments, from B to C and from A to C as AB , BC and AC
measurements, formulations, interpretations, respectively. Also, as an example, assume that the
applications and predicting possible outcomes. Here length of the line segment AC is 15 m. Then,
  
naturally arise some important questions: How to AB  BC  AC .
represent displacement? How to measure it? How to use 30 m 25 m 15 m
it in describing the motion of a body? How to relate it Here emerges an entirely different way of addition: a
to other kinematical quantities? displacement in one direction when added to a
In the example of the strolling girl described above displacement in another direction gives a displacement
(Fig. 4.1), her position changes from point A to point in a third direction whose magnitude may be different
B (see the dotted line). The change in the position can from the sum of the magnitudes of the displacements
be represented by an arrow placed between points A added. Look at the arrangement of the arrows along the
and B, the tail of the arrow at point A, representing the sides of the ABC in the figure carefully. This law of
initial position, and its head at point B, the final addition of physical quantities, called the triangle law
position. or parallelogram law (as explained in Chapter 2), which
The length of the arrow tells by how much did the is followed by displacement and many other physical
position change, that is, the value (more formally called quantities is the vector law of addition; and the
the magnitude) of the displacement and the tail-head quantities that follow this law are christened vectors.
orientation indicates the direction in which this change We shall now learn how to represent and work with
occurred. Displacement has a direction also - it is in the displacements in a systematic way. In our example
direction in which the head of the arrow points. (Fig. 4.1), the position of the girl changed from point A
Hence, the displacement of a moving body must tell to B. It makes sense to represent her position, and also
you two things: by how much did the position change the change in position in a more specific, mathematical
and in which direction did the change occur. Only the way. A convenient way to do it is to draw a position
value of displacement or only its direction is not vector from a reference point.

enough. In a two dimensional plane the position vector r of a
Before we proceed further, we must understand another point whose coordinates are (x, y) is represented as
characteristics of this quantity. Suppose the girl strolling 
r  xiˆ  yjˆ, as shown in Fig. 4.3.
on the ground goes from point A to point B and then
The Cartesian coordinates x and y are called the
from point B to point C , as depicted in Fig. 4.2.

scalar components of the position vector r , and iˆ and

67
ĵ are unit vectors in the direction of x- and y- axes direction? Which direction should be ascribed to

respectively. average velocity? The direction of average velocity vav

is that of r , which is directed along a chord across the
y
path shown in the above figure.
(x, y)
The concept of average velocity has limited application
 as it lacks in details of motion. Many times we are
r interested in velocity of a particle when it is at a certain
position or at an instant of time. This necessitates
developing the concept of instantaneous velocity. While
 calculating average velocity the shorter the intervals we
x
O choose, the smaller is the difference between the
Fig. 4.3 corresponding small segment of the path and its chord.
For a sufficiently small path length, the chord will be
We can also represent the position of a point by r (length practically indistinguishable from the tangent drawn at
of the line drawn from origin to the point) and  (angle the any point of this segment of the path. The direction of
line makes, say, with x- axis in anticlockwise sense). The instantaneous velocity is the direction of the tangent at
two descriptions of the position vector are equivalent in the point of the path where the moving point is at a
the sense we can pass back and forth between them. If we given instant of time.
are given ( x, y ) , we can find r and  from The instantaneous velocity is

y  r
r  x 2  y 2 and tan   , v  lim
x t 0 t

and in case (r , ) are known, we can obtain x and y which in the notation of calculus is written as

 dr
from v . …(4.2)
x  r cos , y  r sin . dt
If a point moves from position A whose position vector We can express instantaneous velocity in terms of its
  components as
is r1 to position B with position vector r2 , as shown in 
Fig. 4.4, the displacement, that is, the change in v  vx iˆ  v y ˆj ,
position, is dx dy 
   where vx  , and v y  . The direction of v is
r  r2  r1  x iˆ  y ˆj. dt dt
y along the tangent to the path, see Fig. 4.5.
Path
y
 B
A r B
 
(t + t)
r1 r2 
x r
O A
t
Fig. 4.4 x
O
The following should help you to draw the direction of
  Fig. 4.5
r correctly: r is the vector that must be added to
 An important point to be noted is that the instantaneous
the initial position vector r1 to give the final position 
    velocity v is directed along the tangent to the path, but
vector r2 , that is, r1  r  r2 .
its magnitude is not the slope of that line. Why? Notice
Now we shall correlate the displacement of a motion to that the diagram is not a position-time graph.
the corresponding time interval which leads to the 
Consequently, the magnitude of v , the instantaneous
concept of velocity. As in Chapter - 3, the average
velocity is defined as the ratio of the displacement to speed, is not given by the slope of the tangent.
the time interval over which the displacement occurred. For a point moving in one dimension, a change in velocity
   can be effected only be changing its value; it can be
 r  r r
vav  2 1  . …(4.1) increased or decreased. When the velocity of the point
t2  t1 t reverses its sign, the direction of motion of the point is
It can be readily seen that the magnitude of the average reversed. In case of motion in two dimensions, as well as
velocity is obtained by dividing the magnitude of in three-dimensional space, there arise many possibilities.
displacement by the time interval. What about its

68
For a point in planar motion, its velocity may change dvx dv y
only in magnitude or only in direction or in most of the where a x  , and a y  .
dt dt
cases, in both magnitude and direction.
In passing it is worth noting that it is far more
Thus, in a curvilinear motion the velocity continuously
changes, which means that the point moves with an convenient to calculate the two components a x and a y
acceleration. To determine this acceleration (its with the help of calculus or otherwise than calculating

magnitude and direction), we have to find the change of vector a.
velocity as a vector, i.e., we have to determine the 
Can we determine acceleration a directly from the path
change of the magnitude of velocity and the direction of of the point? No. We need to know how each
change in velocity. component of the velocity varies as a function of space

Suppose that a point has a velocity v1 at an instant of and time. Figure 4.7 shows possible directions for the

time t1 and v2 after a time interval t, that is to say at acceleration of a point that travels along a curved path
with varying speed.
instant t2 . The change in velocity is obtained by
  y
subtracting vector v1 from v2 . The average
a
acceleration is the ratio of the velocity change to the
interval of time t over which this change occurs. Path of point
   a
 v v v
aav  2 1  . …(4.3) a
t2  t1 t
 x
The direction of average acceleration aav coincides

with the direction of the change in velocity vector v . Fig. 4.7

Does the direction of v coincide with the direction of 
  Example 1. The position vector r of a particle is
v2 ? Can it? What if the point starts from rest? 
The direction of acceleration of a point moving in a given by the following equation r (t )  t 3iˆ   t 2 ˆj ,
plane does not coincide, in general, with the direction 10
of velocity. In order to find the direction of where   ms 3 and   5 ms 2 . At t = 1 s, what
3
acceleration, we compare the directions of the velocities are the velocity and acceleration of the particle?
at two close points on the path. Since the velocities are The derivative of the position vector function gives the
directed along the tangents to the paths, the direction of velocity vector and the derivative of the velocity vector
acceleration can be determined from the shape of the function gives the acceleration, (Eqs. (4.2) and (4.4)).
 
path. Since the change v2  v1 of the velocities at two 
v (t )  3t 2iˆ  2 t ˆj
close points is always directed towards the bending of 
the path, it means that acceleration is always directed a (t )  6t iˆ  2 ˆj .
inside of the curved path, see Fig. 4.6. Substituting the given values of  and , and t  1 s
v1 into the expressions for the position vector, velocity and
acceleration, we obtain

v1  10
r  iˆ  5 ˆj m
  3
v v2 
v  10iˆ  10 ˆj m/s
Fig. 4.6 
a  20iˆ  10 ˆj m/s 2 .
Since average accelerations of a motion computed over  
different time intervals are not necessarily same, it  At t  1 s, what is the angle between r and v ?
   
warrants to develop the concept of instantaneous Between r and a ? Between v and a ?
acceleration: acceleration at a certain point of the path We can use the dot product of two vectors (Eq. 2.4,
or acceleration at a certain instant of time. Chapter 2) to compute these angles. But in this problem
By choosing a sufficiently small t, we arrive at the you know the x- and y- components of the position
concept of instantaneous acceleration. The velocity and acceleration vectors. You can find the
instantaneous acceleration is the rate of change of required angles straightaway. All you need to do is to
velocity relative to time: draw these vectors on x-y plane (Fig. 4.8), and use

 dv elementary trigonometry.
a  ax iˆ  a y ˆj …(4.4)
dt

69
y   1
v a y  y0  u y t  a y t 2 …(4.6c)
10 m/s 2
10 m/s2
v 2y  u y2  2a y S y . …(4.6d)
10 m/s 20 m/s2
Now the most important question is how the two sets of
5m equations are related? The answer to this question lies

r in that the time parameter t is the same for each, since t
x represents the time at which the point occupied a
O
10
m
position defined by the co-ordinates x and y.
3 The equations of motion in a plane may also be
Fig. 4.8  expressed in vector form. For example, velocity of the
point at a variable time t can be written in terms of its
components as
4.3. Motion in a Plane with Constant 
v  vx iˆ  v y ˆj
Acceleration
Let us consider the case of two dimensional motion of a  (u x  ax t )iˆ  (u y  a y t ) ˆj
point in which the acceleration does not vary either in  (u x iˆ  u y ˆj )  (a x iˆ  a y ˆj ) t.
magnitude or in direction. In that case the components
 The first quantity in parentheses is the initial velocity
of acceleration a in any Cartesian coordinate system 
will not vary, that is, ax = constant and a y = constant. vector u and the second is the (constant) acceleration

Under this condition the motion of the point can be vector a multiplied by t. Thus, the vector relation
  
described as the sum of two component motions v  u  at …(4.7)
occurring simultaneously with constant acceleration is equivalent to the two scalar relations. This relation

along each of the two axes. This simplifies the analysis shows that the velocity v at time t is the sum of the
amazingly. The point will move, in general, along a 
initial velocity u and the (vector) change in velocity,
curved path in the plane. Will the point move on a 
at , during the time interval t under the constant
curved path even if one component of the acceleration, 
acceleration a. Similarly, the vector equation for the
say a x, is zero? Can one component of the velocity, say
displacement is
vx, have a constant, non-zero value? What about the
  1
motion of a cricket ball which follows a curved path in S  ut  at 2 . …(4.8)
a vertical plane when the effect of air resistance is 2
neglected? One can easily interpret this equation by ascribing
We can obtain the general equations for two meaning to the two terms that appear on its right hand
dimensional motions with constant acceleration simply side.
by setting We shall now apply the concepts discussed above to
ax = constant and a y = constant. some common motions, viz. projectile motion, circular
The equations for constant acceleration (Eqs. 3.7, motion, relative motions in two dimensions, and so on.
through 3.10, Chapter 3) then apply separately and
independently to both the x- and y- components of the
 4.4. Projectile Motion

displacement vector S , the velocity vector v , and the The fact that the laws of natural sciences are non-intuitive in nature is
 often reflected in our everyday life. For example, for the motion of a
acceleration vector a. From this idea we generate two ball thrown in air, many of us believe that the force used to throw the
sets of equations given below. ball up somehow stays with it. The ‘force of the hand’ is supposed to
(i) Equations for motion in the direction of x- axis: be gradually overcome by the force of gravity, which ultimately
causes the ball to fall. It is not surprising that as late as the sixteenth
vx  u x  a x t …(4.5a) century it was believed that when a shell was fired, it was given an
1 ‘impressed force’ that produced ‘violent’ motion in a straight line,
S x  u x t  ax t 2 …(4.5b) thereafter followed a region of mixed motion (‘violent’ plus ‘natural’
2 motion vertically down) because of air resistance, and finally, the
1 ‘natural’ motion vertically down prevailed, see Fig. 4.9. Initially,
x  x0  u x t  ax t 2 …(4.5c) Galileo also believed that the motion of a projectile was governed by
2 an ‘impressed force’ that gradually diminished. It was only after he
vx2  u x2  2ax S x . …(4.5d) had developed his principle of inertia that he could tackle the problem
of projectile motion properly.
(ii) Equations for motion in the direction of y- axis: Galileo had arrived at the crucial insight that a projectile near the
vy  u y  a yt …(4.6a) surface of the earth has two independent motions: A horizontal
motion at constant speed and a vertical motion subject to the
1 2 acceleration due to gravity.
S y  u yt  at …(4.6b)
2 y

70
Mixed
made immediately. We see that vx remains constant in
Violent time and is equal to the initial x- component of velocity,
Natural
since there is no horizontal component of acceleration.
Also, for the motion in y- direction, we note that
expressions for v y and y are identical to that for v y and
y for a particle thrown vertically upward and moving
with constant acceleration g, as discussed in
Section 3.5.2, Chapter 3.
If we solve for t in Eq. 4.11 and substitute the
expression for t into Eq. 4.12, we find the equation of
Fig. 4.9
the trajectory:
This very common form of motion is surprisingly  g  2
y  (tan ) x   2
 2v cos2  
simple to analyze if two facts are taken into x , …(4.13)
consideration: (i) The acceleration due to gravity g is  0 
constant over the range of motion and is directed 
which is valid for the angle  in the range 0    .
downward. (ii) The effect of air resistance is negligible, 2
which makes us to assume that there is no acceleration Why? This equation is of the form y  ax  bx , which
2
in the horizontal direction.
If we choose our reference frame such that the is the equation of a parabola that passes through the
y- direction is vertical and positive upward, then a y = –g origin. You will learn about this function when you
(as in one-dimensional free fall) and ax = 0 (since air study quadratic equations in algebra or parabola in
resistance is neglected). Suppose that at t = 0 the coordinate geometry.
projectile leaves the origin (0, 0) with a velocity v0, as It is extremely useful to investigate the motion of the
projectile in a little detail and compute a few variables,
in Fig. 4.10. If the velocity vector makes an angle  
(angle of projection) with the horizontal, the initial x- like its velocity v as a function of time, the angle its
and y- components of velocity are given by velocity vector makes with the horizontal, its speed, its
v0 x  v0 cos  and v0 y  v0 sin . position vector and distance from the origin, the angle
between its velocity vector and acceleration vector at
Substituting these expressions into Eqs. 4.5 and 4.6 certain instant t , and so on.
with a x = 0 and a y   g gives the velocity 
We can obtain the velocity v as a function of time for
components and coordinates for the projectile at any the projectile by noting that Eqs. 4.9 and 4.10 give the
time t. x- and y- components of velocity at any instant:

v  (v0 cos )iˆ  (v0 sin   gt ) ˆj.
Also, since the velocity vector is tangent to the path at

y any instant, as shown in Fig. 4.10, the angle  that v
g makes with the horizontal can be obtained from vx and
vy = 0
vy v0x v y through the expression

v0
v0x vy
v0y tan   .
vx
v0x
 x
Further, by definition, the instantaneous speed v is
O v0x  
equal to the magnitude of the instantaneous velocity v ,
therefore,
v0
–v0y
v  vx2  v 2y .
Fig. 4.10 The expression for the position vector as a function of
vx  v0 cos  …(4.9) time t for the projectile follows directly from Eq. 4.8,
 
v y  v0 sin   gt …(4.10) with a  g we have

x  (v0 cos )t   1


…(4.11) r  v0 t  gt 2 .
2
1
y  (v0 sin )t  gt 2 . …(4.12) This expression is plotted in Fig. 4.11. It is interesting
2 to note that the motion can be considered as the
By a look at these equations certain observations can be

71

superposition of the term v0 t , which is the acceleration vertically down. In time t  x / v0 x , the
displacement if no acceleration were present, and the bullet reaches a height given by
1 1 1
term gt 2 , which arises from the acceleration due to y  v0 y t  gt 2  H  gt 2 .
2 2 2
gravity. In other words, if there were no gravitational 1 2
acceleration the projectile would continue to move This is lower than H by gt , which is just the amount
 2
along a straight path in the direction of v0 . the koala falls in this time. For large v0 the koala is hit
y very near its original height, and for small v0 it is hit
12 just before it reaches the ground. 
 2
gt
v0 t
(x, y) 4.4.1. The Maximum Height and
 Horizontal Range of a Projectile
r
O
x There are two points that are of special interest: the
highest point of the trajectory with Cartesian
Fig. 4.11 coordinates (R/2, H) and the point with coordinates
(R, 0) where the projectile comes in level with the
We conclude that projectile motion is the superposition launch point, see Fig. 4.13. The distance H is called the
of two motions: (1) the motion of a freely falling body maximum height of the projectile and R its horizontal
in the vertical direction with constant acceleration and range.
(2) motion in the horizontal direction with constant
velocity. y
R 
 , H  , vy  0
 Example 2. A hunter wishes to shoot a koala 2 
hanging from a branch. The hunter aims right at the t'
v0
koala, not realizing that the bullet will follow a
parabolic path and thus will fall below the koala. The h
koala, however, seeing the firing, lets go of the branch  x
and drops out of the tree, expecting to avoid a hit. Show t=0 t=T
that the koala will be hit regardless of the initial R
velocity of the bullet so long as it is large enough to
travel the horizontal distance to the tree before hitting Fig. 4.13
the ground.
Let the horizontal distance to the tree be x and the We can determine the maximum height H reached by
original height of the koala be H, as shown in Fig. 4.12. the projectile by noting that v y  0 at the peak.
Then the gun is aimed at an angle given by tan  = H/x. Therefore, Eq. 4.10 can be used to determine the time t
If there were no gravity, the bullet would reach the it takes to reach the peak:
height H (with vertical velocity v0 y ) in the time t taken v sin 
t  0 .
for it to travel the horizontal distance x, that is, g
Substituting this expression for t  into Eq. 4.12 gives
the maximum height H in terms of v0 and :
2
1
gt 2  v sin   1  v0 sin  
2
H  v0 sin   0  g 
H  v0 y t  g  2  g 
v0 y v02 sin 2 
v0 y or H . …(4.14)
x 2g
v0 x
Alternatively, the maximum height H can be obtained
from Eq. 4.6d directly,
Fig. 4.12 02  (v0 sin ) 2  2 gH ,
x v02 sin 2 
v0 y t  H in time t 
. whence H  .
v0 x 2g
However, because of gravity, the bullet has an The range R is the horizontal distance traveled in

72
twice the time the particle takes to reach the peak, that addition, for any  other than 45, a point with
is, in time 2t . Since the acceleration in the vertical coordinates (R, 0) can be reached with two
direction is constant, the time of decent of the projectile complementary values of , such as 75 and 15°. Of
must equal the time of ascent, therefore, the total time course, the maximum height and time of flight will be
of flight is equal to different for these two values of . Figure 4.14
2v sin  illustrates, by way of example, that for two
T  2t   0 . …(4.15) complementary angles, 75 and 15, and 60 and 30,
g
This can also be seen that by setting y = 0 in Eq. 4.12 the range of projectile is same. We used g  9.8 m/s 2 to
and solving the quadratic equation for t, one solution is generate the trajectories of the projectile launched at
2v sin  different angles.
t  0, and the other solution is t  0  T . Using
g To emphasize this fact let us consider another example.
Eq. 4.11 and noting that x  R at t  T , we find that If v0  20 m/s and R  30 m, then
 2v sin   2v02 sin  cos  Rg (30 m)  (9.8 m/s 2 )
R  v0 cos   0   sin 2    0.735 .
 g  g v02 (20 m/s)2
which is Thus   23.7  or 66.3 . Notice that   45  ,
sin 2
v02 where   21.3 , see Fig. 4.15.
R . …(4.16)
g
y
It is important to note that Eq. 4.16 is valid only when
the projectile returns to the initial vertical level, that is,
y  0.
Keep in mind that Eqs. 4.14 and 4.16 are useful only for 

calculating H and R if v0 and  are known and only for a
45
symmetric path, as shown in Fig. 4.10. The general O x
expressions given by Eqs. 4.9 through 4.12 are the most
important results, since they give the coordinates and Fig. 4.15
velocity components of the projectile at any time t.  Find the maximum height reached and horizontal
You should also note that the maximum value of R range if a projectile is launched at 50 m/s at an angle
from Eq. 4.16 is Rmax  v02 / g . This result follows from (i) 15, (ii) 30, (iii) 45, (iv) 60 and (v) 75 with the
the fact that the maximum value of sin 2 is unity, horizontal. Take g  9.8 m/s 2 .
which occurs when 2 = 90°. Therefore, we see that the An important practical aspect of the motion of the projectiles must be
range R is a maximum when the angle of projection understood. Computations based on the above equations are accurate
 = 45, if resistance of air is neglected. only when air resistance can be neglected and the acceleration due to
gravity is constant both in magnitude and in direction.
Figure 4.14 illustrates various trajectories for a The predictions of these equations are valid only when the projectile
projectile of a given initial speed. is launched at a speed much less than its terminal speed (refer to
Chapter 16). They do not really apply even to such commonplace
projectiles as cricket and golf balls, let alone arrows, bullets, or
y (m)
ballistic missiles. They may be applied to slower projectiles, such as
150 a shot-put. Nonetheless, in other cases they do provide a good first
. approximation to a complete, and usually far more complex solution.
75 v0 = 50 m/s
The student should not feel compelled to memorize the
100 60 expressions for time of flight, maximum height, and
range or the equation of trajectory. These expressions
45
are useful only in answering straightforward questions.
50 30 The important thing is to follow the line of reasoning
r 15 and steps used to obtain these results. The wisest way to
x (m) handle problems on projectile motion is NOT TO USE
O
THESE FORMULAE, EVER. You are not supposed to
50 100 150 200 250 even remember them. You can anyway derive any of
them in 8 to 10 seconds if need be.
Fig. 4.14 It is far more fruitful to solve problems on projectile
from the basics: draw a diagram, resolve the initial
As you can see, the range is maximum for  = 45°. In velocity into horizontal and vertical components, and

73
then analyze the two components of motion separately y
A
and independently, using the familiar equations
20 m/s
1 16 m/s
v  u  at , S  ut  at 2 , v 2  u 2  2aS for motion
2
with constant acceleration. This makes the solution 53 x
unimaginably simple. O 12 m/s B
 Example 3. A projectile is launched at moment t  0
Fig. 4.17
from point O on the ground with a velocity 20 m/s at an
angle of 53° with the horizontal. Point O is origin of a T  2t A  2  1.6 s  3.2 s.
coordinate system whose x- axis is horizontal and y- axis
is directed vertically upward, see Fig. 4.16. Take the unit  Can you find T by using Eq. 4.12?
vectors along the x- and y- axes as iˆ and ĵ respectively. (b) The projectile lands on the ground at point B, see
Take the acceleration due to gravity as 10 m/s2 downward,
Fig. 4.17. What are the coordinates of point B?
that is, opposite to positive y- axis. Take sin 53° = 0.8, On multiplying the horizontal component of velocity
cos 53° = 0.6. vx (  v0 x ) by the time of flight T , we obtain the x-
y coordinate of the point where the projectile falls.
A
R  xB  (12 m/s)  (3.2 s)  38.4 m.
20 m/s
Thus, the coordinates of point B are (38.4, 0) m.
We could have calculated the range of the projectile by
53 x directly using the Formula 4.16. But it must be borne in
O B mind that a line of reasoning/approach is more
important than a formula.
Fig. 4.16
(c) What is the maximum height above the ground
(a) What is the total time of flight of the projectile? attained by the projectile during the course of its flight?
You can calculate the time of flight directly from Formula 4.12 gives the variation of y- coordinate with
Formula 4.15 as time. Substituting into this formula t A  1.6 s for t , we
2v sin  2  20 m/s  sin 53 obtain the y- coordinate corresponding to the apex A
T 0   3.2 s.
g 10 m/s 2 of the trajectory, which is the height H to which the
projectile ascends.
But as instructed above, you should follow the line of
1
reasoning that was employed in deriving this and other H  (16 m/s)  (1.6 s)   (10 m/s2 )  (1.6 s) 2  12.8 m.
formulas of projectile motion to solve even simple 2
straightforward problems. You will understand the You can verify that Formula 4.14 gives the same result.
benefits of this habit when, after solving a variety of (d) At the moment t = 1 s, what is the y- coordinate of
problems, you attain maturity and realize how the projectile?
beautifully this approach works for most complex Substituting t  1s and the values of other quantities
problems.
into Formula 4.12 you get the y- coordinate of the
The x- and y- components of the initial velocity
projectile as
of the projectile are (20 m/s) cos 53  12 m/s and 1
y  (16 m/s)  (1 s)  (10 m/s 2 )  (1 s)2  11 m.
(20 m/s) sin 53  16 m/s, see Fig. 4.17. 2
At the apex A of the trajectory, the velocity has only (e) What is the equation of the path followed by the
horizontal component, while v y vanishes. In order to projectile?
find the instant t A at which the projectile reaches point Coordinates of the point where the projectile is at time t
are
A, we substitute tA for t into Formula 4.10 and equate
x  12t
the obtained result to zero.
1
(16 m/s)  (10 m/s 2 )t A  0, and y  16t  (10) t 2 .
2
which gives t A  1.6 s. x
Since the point from where the projectile was launched Substituting t  from the first equation into the second
12
and the point on the ground where it lands are at the we obtain the equation of the trajectory
same horizontal level, the time of flight is equal to 2t A . 4 5 2
y  x x .
3 144

74
(f) At what moment of time the height of the projectile vx 12 m/s
tan     2, 6 m/s
above the ground is 11 m? vy 6 m/s 
We substitute y  11 m into Eq. 4.12 for obtaining the
or   tan 1 2. 12 m/s
required time:
1 (i) At what moment of time the velocity of the
11 m  (16 m/s)t  (10 m/s2 )t 2 , projectile is parallel to x- axis?
2 At the apex of the trajectory velocity has only
which gives t  1 s and 2.2 s.
horizontal component, the vertical component v y
For both upward and downward motions the
acceleration of the projectile is same, the time it takes vanishes. The projectile reaches the apex at t  1.6 s;
from 11-m height to the apex is same as the time of thus, at this moment its velocity is parallel to x- axis.
motion from the apex to a height of 11 m again. It (j) At what moment(s) of time the x- component of
follows that the points are symmetrical about the apex velocity of projectile is twice the y- component in
A, which means that the trajectory is symmetrical magnitude?
about point A, or more specifically, about the vertical This question was indirectly answered in Parts (g).
line from the apex A. However, alternatively, we can express the x- and y-
components of velocity at moment t as
(g) At the moments the y- component of the
displacement of the projectile is 11 m, what is its vx  12 m/s and v y  (16  10t ) m/s.
y- component of velocity? The x- component of velocity is twice of the
In the previous problem we computed the moments of y- component in magnitude if
time at which the height of the projectile above the 12  2 |16  10t |
ground is 11 m. Using Eq. 4.10 we obtain the velocity  12  2(16  10t ), which gives t  1 s,
of the projectile at these two moments, and 12  2(16  10t ), which gives t  2.2 s .
at t  1 s, v y  16 m/s  10 m/s 2 1 s  6 m/s,
(k) At time t (t > 0), the coordinates of the projectile are
at t  2.2 s, v y  16 m/s  10 m/s2  2.2 s  6 m/s. (x, y). Find whether there is a value of t for which
The velocity of the projectile when it is at a height of y  2 x.
11 m from the launch point can also be calculated from It can be shown that in the interval 0  t  3.2 s, there
Eq. 4.6d as follows: set y  11 m, a y  10 m/s2 and is no instant of time at which the y- coordinate is twice
as large as x- coordinate. Using simple algebra we can
v0 y  16 m/s in this equation - 1
show that the equation y  2 x, i.e., 16t  t 2  12t
v y2  (16 m/s) 2  2(10 m/s 2 )  (11 m), 2
does not have any real solution in the interval
which yields v y   6 m/s, as expected. These
(0, 3.2 s].
velocities are shown in Fig. 4.18.
(l) At the moment of time t1 (t1 > 0) the x- component of
y 6 m/s displacement of the projectile is twice of its
12 m/s y- component of displacement. What is the value of t1?
20 m/s 12 m/s Now we are so comfortable with formulas and
6 m/s formulations that the best strategy comes to our mind
11 m 11 m immediately. To arrive at the answer of this question
53 x
O B we simply write
 1 
Fig. 4.18 12t1  2  16t1  10t12 
 2 
(h) At the moment the y- component of displacement of which gives t1  2.0 s.
the projectile is 11 m, what is the angle between its
velocity and y- component of velocity? (m) At the moment the x- component of the
It suffices to know the two components vx and vy of the displacement of the projectile is twice of its
velocity at a moment (or a position) to compute the y- component of displacement, what is the angle
angle between the velocity and its y- component. We between the x- component of velocity and velocity?
calculated the y- component of velocity at y  11 m in In the previous problem we calculated the moment of
the previous problem. The required angle can be time at which the x- component of the displacement is
obtained from the velocity diagram. twice the y- component of the displacement. At this

75
moment of time (t  2.0s) , the x- and y- components of the velocity of separation decreases (can you show it?), but
the velocity of the projectile are it never reduces to zero in this case. Throughout the
motion the projectile has a component of velocity directed
v x  12 m/s and v y  16 m/s  (10 m/s 2 )  (2 s)  4 m/s,
away from the thrower. Consequently, it moves away
that is, 4 m/s in downward direction. from him until it hits the ground.
Here, the angle that is sought is given by If the velocity vector were to become perpendicular to the
position vector at some point in the trajectory, then, in the
4 m/s 1
tan     12 m/s subsequent motion a component of velocity would be
12 m/s 3 directed inside toward the point of projection, the
1 projectile would be coming closer to the thrower. This
or   tan 1   . 4 m/s
situation does not arise in this example. The projectile
 3
always moves away from the thrower.
(n) Show that there is no instant of time t (t > 0) at which the
position vector of the projectile is perpendicular to its velocity  What is the maximum angle to the horizontal at
vector. which the projectile can be thrown and always be
From the two right angle triangles in Fig. 4.19, moving away from the thrower? See Example 6.
y (o) During its course of flight the projectile just clears
two parallel walls each of height 11 m. What is the
t 12 m/s separation between the walls?
20  As computed in Part (f), the projectile is at a height of
16 m/s
10t – 16
11 m at the moments t  1 s and t  2.2 s .
y = 16t – 5t2
t=0  x These are the moments at which the projectile clears the
O 12 m/s x = 12t B 11-m high walls, one after the other, Fig. 4.20. On
multiplying vx by the time interval t  2.2 s  1.0 s
Fig. 4.19  1.2 s the projectile takes in clearing the walls, we
y v 16t  5t 2 12 obtain the separation between them
 x   , x  vx t  12 m/s  1.2 s  14.4 m.
x v y 12t 10t  16
which simplifies to 5t 2  24t  40  0. y
t=1s t = 2.2 s
If the velocity vector of the projectile is perpendicular to 20 m/s
its position vector at certain instant of time, the scalar
product of these two vectors must equal zero at this 11 m 11 m
instant. Figure 4.19 has been constructed under the 53 x

assumption that the velocity of the projectile is O x B
perpendicular to its displacement at an instant t . We have
Fig. 4.20
[12iˆ  (16  10t ) ˆj ]  [12t iˆ  (16t  5t 2 ) ˆj ]  0,
(p) At what moment of time t the velocity of the
which yields the same quadratic as above.
projectile is perpendicular to its initial velocity?
Further, if we represent by v̂0 the unit vector in the Figure 4.21 solves this problem immediately.
direction of initial velocity vector, then we can
  y
vectorially express the condition v  r as
90
  1 
(20vˆ0  gt )   20vˆ0 t  gt 2   0. t=t
 2  20 m/s 12 m/s
16 m/s
To evaluate the scalar product in the above equation, v
 37
note that the angle between unit vector v̂0 and g is
t=0 53 90 37 x
90    90  53. The same quadratic again! 12 m/s x = 12t

B
O
The quadratic equation 5t 2  24t  40  0 does not
Fig. 4.21
have any real solution. This implies that during the
flight, the velocity of the projectile is never From the triangle of velocities at time t we see that
perpendicular to its position vector. v
A very fascinating implication of this result is that the tan 37  , which gives v  9 m/s  v y  9 m/s,
12
projectile always moves away from the thrower. At the
whence 9 m/s  16 m/s  (10 m/s2 )t , giving t  2.5 s.
moment of projection the projectile has a velocity of
20 m/s directed away from the thrower. As time passes, We can approach this problem in many other ways.

76
(i) (12i  (16  10t ) ˆj )  (12iˆ  16 ˆj )  0. Expand the dot at the end of the time interval. At the initial moment
product to obtain t  25 s. t  0 , the velocity of the projectile is (12iˆ  16 ˆj ) m/s
 and at the moment t  1.6 s, its velocity is 12i m/s .
(ii) (20vˆ0  g )  (20vˆ0 )  0, where v̂0 is the unit vector
in the direction of initial velocity. Clearly, the angle Substituting these values in the above formula we
 obtain the average velocity of this time interval,
between v̂0 and g is 90  53. Expand the dot
 (12iˆ  16 ˆj )m/s  (12iˆ ) m/s
product for answer. vav   (12iˆ  8 ˆj ) m/s.
(iii) The choice of the coordinate axes shown in Fig. 4.22 2
turns out to be extremely convenient. Let the velocity of We shall now calculate the average velocity using its
the projectile be perpendicular to its initial velocity at definition: it is the displacement divided by the time
instant t. The x- components of velocity at this moment interval in which the displacement was undergone.
must be equal to zero. From the solution of Parts (b) and (c), we can see that
the displacement of the projectile in the time interval

x t  0 to t  1.6 s is r  (19.2iˆ  12.8 ˆj ) m. (How?)
Therefore, the required average velocity is
6 m/s2 ax = –8 m/s2
8 m/s2 53  (19.2iˆ  12.8 ˆj ) m
10 m/s 2 ay = –6 m/s2 vav   (12iˆ  8 ˆj ) m/s.
t 1.6 s
20 m/s

y (t) If vav represents the average speed and | vav | the
53 magnitude of the average velocity for the time interval

O (v0x = 20 m/s, v0y = 0) t = 0 to t = 1.6 s, show that vav  | vav | .
Fig. 4.22 It is obvious that the length of the curve (parabola) is
greater than the length r of the chord in Fig. 4.23.
Apply vx  u x  a x t to the motion of the projectile - Hence the average speed vav is larger than magnitude

0  (20 m/s)  (8m/s 2 )t , of average velocity | vav | . This statement is true for
which gives t  2.5s. any time interval.
(q) What is the change in the velocity of the projectile y
s
in the time interval t = 0 to t = 1.6 s?
In case the acceleration of a body is constant, the

change in velocity v in time interval t is related to r
  
the (constant) acceleration a as v  a t. Hence, x

v  ( 10 ˆj m/s 2 )  (1.6 s  0)  16 ˆj m/s. O

   Fig. 4.23
 Use v  v2  v1 to arrive at the answer.
(u) A girl located on x- axis 60 m away from the point
(r) What is the average acceleration of the projectile in of projection starts running at the moment the projectile
the time interval t = 0 to t = 1.6 s? is thrown. How fast, and in which direction must she
In Section 3.3.2, Chapter 3 we showed that for a run in order to catch the projectile at the level from
uniformly accelerated motion the average acceleration which it was thrown, if she runs at a constant speed?
over any interval of time is same and equal to the We shall use the solution of Parts (a) and (b). To catch
instantaneous acceleration. Therefore, the average the projectile at the same level from which it was
acceleration of the projectile in the time interval t  0 thrown the girl must cover a distance of
to t  1.6 s is 10 m/s 2 ˆj. 60 m  38.4 m  21.6 m in 3.2 seconds. Hence she
You can also use the definition of average acceleration must run at a speed of 21.6 m / 3.2 s  6.75 m/s
and find the answer. towards the point of projection.
(s) In the time interval t = 0 to t = 1.6 s, what is the (v) A catcher is standing on x- axis 36 m away from the
average velocity of the projectile? point of projection, see Fig. 4.24. How far above its
For a uniformly accelerated motion the average velocity initial level is the projectile caught?
in any time interval is Projectile reaches the catcher in 36 m / (12 m/s)  3 s.
 
 u v At this moment of time the projectile is at a height of
vav  ,
2 1
  h  (16 m/s)  (3 s)   (10 m/s2 )  (3 s) 2  3 m.
where u and v are the velocities at the beginning and 2

77
Either the catcher is really tall or good at jumping! 3
velocity diagram we get tan   , which gives
y 4
  37. Therefore,
20 m/s
4
a N  g cos   10 m/s 2   8 m/s 2
5
53 h
x 3
36 m
and a   g sin   10 m/s 2   6 m/s 2 .
O 38.4 m 5
Fig. 4.24 Pay attention to the fact that the tangential acceleration
is negative at the given moment. Does the tangential
(w) The catcher in Part (v) sees the projectile when it is acceleration of the projectile have a negative value at
right above his head and decides to run and catch it all positions during its motion?
before it falls on the ground. If his reaction time is 0.5
(z) At what moment of time the magnitude of tangential
s, will he succeed?
No. The projectile comes to the initial level with launch component of acceleration of the projectile is
maximum?
point in 3.2 s  3 s  0.2 s, whereas the catcher requires
We observe that the angle  in Fig. 4.25 assumes its
0.5 s to start running. maximum value at moments t  0 and t  3.2 s .
(x) At what moment of time is the speed of the Hence, obviously, these are the moments at which the
projectile 15 m/s? tangential acceleration of the projectile will have its
maximum value. See solution of Part (y)
The formula vx2  v2y  v gives
At what moment of time the normal component of the
acceleration of the projectile has its maximum value?
(12 m/s) 2  (16 m/s  (10 m/s2 ) t ) 2  15,
This happens when the angle  (Fig. 4.25) is the least.
or t  0.7 s, 2.5 s. At the apex of the trajectory, the velocity of the
(y) At the instant t = 0.7 s, what are the normal and projectile is perpendicular to its acceleration, the radial
tangential components of the acceleration of the acceleration becomes equal to the acceleration due to
projectile? gravity g  10 m/s 2 .
The normal component of acceleration is the At what moment(s) of time the magnitude of the normal
component that is perpendicular to the velocity vector, acceleration is twice the magnitude of the tangential
while the tangential component is in the direction of acceleration?
velocity vector. You are advised to revisit this problem We shall correlate the acceleration diagram to the
after going through Section 4.5.1. velocity diagram to obtain the vertical components of
At every moment of time during the flight the the velocity at the moment the given condition is
acceleration of the projectile is in the downward satisfied, see Fig. 4.26. Since a N  2 | a | we have
direction and is equal to the acceleration due to gravity
1
g . As shown in Fig. 4.25, the normal and tangential g cos    2 g sin  or tan    . The velocity
2
components of the acceleration are aN  g cos  and
diagram gives
a   g sin . v y  vx tan    6 m/s.
y Now substituting this value of v y into Eq. 4.10, we obtain
9 m/s 
v
12 m/s 6 m/s  16 m/s  ( 10 m/s 2 )t1 or t1  1 s
16 m/s a  aN
and 6 m/s  16 m/s  (10 m/s 2 )t2 or t2  2.2 s.
g
 x
y
O
t = 0 12 m/s
vy
Fig. 4.25 
vx
The angle  can be obtained from the velocity diagram. |a|  aN aN  a
At the moment t  0.7 s the horizontal and vertical g g
x
components of velocity are vx  12 m/s and O

v y  16 m/s  10 m/s 2  0.7 s  9 m/s. From the Fig. 4.26

78
Example 4. A particle is projected from a horizontal c  b   b  
2 2

x-z plane such that its velocity vector at time t is given    t      


 2  c   c  
by v  aiˆ  (b  ct ) ˆj. Find (a) time of flight T;
2
(b) maximum height H; (c) range R. b2 c  b 
  t
2c 2  c 
.
(a) The time of flight is the time taken for the vertical
displacement to become zero. We have Clearly, for y to be maximum second term in the above
v y  b  ct b2
expression must be zero. Hence H  .
dy 2c
or  b  ct
dt (c) Range is the horizontal displacement in the time of
t flight. Hence,
or y   (b  ct )dt. 2b/ c 2b / c
2ab
0 R   vx dt   adt  .
If y becomes zero again after time T , 0 0
c
An alternative solution to the whole problem: compare
cT 2 
0  bT  , the given velocity v  aiˆ  (b  ct ) ˆj with the velocity
2
2b of a simple familiar projectile
which gives T  . 
c v  v0 cos iˆ  (v0 sin   gt ) ˆj
(The solution T = 0 corresponds to instant of projection.) term by term -
Alternatively, the vertical velocity becomes zero at v0 cos   a, v0 sin   b and g  c.
b The problem henceforth reduces to the computation of
time given by b  ct  0 or t  , and time of flight is T, H and R in projectile motion.
c
2b Example 5. As shown in Fig. 4.27, a particle is
twice this time, (why?), hence T  .
c projected from point A at angle  to the horizontal
(b) The maximum height is the displacement in where    . Find the speed of projection of the
y- direction at the moment the y- component of velocity particle so that it just grazes the inclined wall?
is zero.
b
We know that v y is zero at t  . So the maximum
c Wall g
height is
2
b cb
b/c
b2
H 0 (b  ct )dt = b  c   2  c   2c .  
Alternatively, the motion of particle is one with O
A
constant acceleration. (Can you see this?) Application l0
of Eq. 4.6d gives
v 2y  u 2y  2a y H Fig. 4.27
or 02  b 2  2( c) H Let the particle be projectile with speed v0 . At the
b2 moment the particle grazes the wall its velocity vector
or H . is parallel to it (why?). Let x0 and y0 be the horizontal
2c
Further, the displacement in the vertical direction in and vertical displacements of the particle at that instant.
time t is
t
ct 2 y
y   (b  ct ) dt  bt 
0
2
c 2b 
  t 2  t  v0 y0
2 c 
c b b b  
2 2

  t 2  2t          O
2   c   c   c   l0 x0

Fig. 4.28

79
From Fig. 4.28, 2 gl 0 sin  cos 
y0 or v0  .
tan   . …(i) sin 2 (  )
l0  x0
In this method the condition    is not needed. Can
Also, the velocity of the particle at time t is
 you reason it out geometrically?
v  (v0 cos )iˆ  (v0 sin   gt ) ˆj. Here is the smartest way to solve this problem.
The time when the velocity vector makes angle  with Choosing a coordinate system wisely does wonders at
the horizontal is given by times.
v sin   gt We choose a coordinate system as shown in Fig. 4.29.
tan   0 Clearly, l0 sin  is the maximum y- displacement if the
v0 cos 
particle grazes wall.
v0 sin(  ) l0 sin 
which gives t  .
g cos  x
At this time, l0 sin
v0 2 sin(   ) cos  y 
x0  (v0 cos )t  ,
g cos  
v0 –
1 g sin  g cos 
and y0  (v0 sin )t  gt 2  
2 O g
l0
v0 2 sin  sin(  ) v0 2 sin 2 (   ) (a) (b)
  . Fig. 4.29
g cos  2 g cos 2 
Substituting v0 y  v0 sin(   ) , v y  0, a y   g cos  ,
Substituting x0 and y0 into Eq. (i) we get
and y0  l0 sin  into Eq. 4.6d we obtain
v0 2 sin  sin(  ) v0 2 sin 2 (  )
 0  v02 sin 2 (  )  2( g cos )l0 sin ,
g cos  2 g cos 2 
tan   ,
v sin(  ) cos 
2
2 gl0 sin  cos 
l0  0 which gives v0  .
g cos  sin 2 (   )
which, after some simple trigonometry, gives Example 6. What is the maximum angle to the
2 gl0 sin  cos  horizontal at which a stone can be thrown and always
v0  .
sin 2 (   ) be moving away from the thrower?
If the stone is projected at a large angle to the
Alternatively, the equation of trajectory of the particle
in the coordinate system shown in Fig. 4.28 is horizontal, say 80 or so, it can be intuitively figured
out or can be inferred by a look at the trajectory
gx 2
y  x tan   (Fig. 4.30) that its distance from the thrower will first
2v0 cos 2 
2
increase and then decrease, attaining a maximum value
and that of the wall projected onto the plane of the in between.
trajectory of the projectile is For some time the stone moves away from the point of
y  (tan  )( x  l0 ) . projection, velocity component on the line connecting
If the particle just grazes the wall, then the points of the point of projection to instantaneous position of the
intersection of the above two curves must coincide. If stone is directed away from the thrower as shown in the
these equations have two distinct solutions, the particle figure. It can be further seen that with the passing of
hits the wall; no real solution implies that the particle time the velocity of separation decreases. At a certain
misses the wall. moment of time the velocity of separation becomes
On eliminating y from the two equations we get zero. This happens at the moment when the velocity of
the stone is perpendicular to its position vector.
 g  2 Thereafter, the velocity component projected on the
 2  x  x(tan   tan )  l0 tan   0.
 2v0 cos   position vector is directed towards the point of
2

projection; the distance of the stone from the thrower


The roots of the above equation coincide if
decreases.
4g
(tan   tan ) 2  l0 tan  The above argument immediately gives us a way to
2v0 2 cos 2  solve the problem: the angle of projection must be such
sin 2 (   ) 2 gl tan  that the velocity of the stone is never perpendicular to
or  20 2 its position vector during its flight, that is, the stone
cos  cos  v0 cos 
2 2

80
must always have a velocity of separation as seen by y
the thrower. vx
v
y v1p v 
v1 v y

v2  x
O
v x
Fig. 4.32
dmax
It can be seen from the figure that at the instant the
80 velocity of the stone is perpendicular to its position
x
O v v
y
vector,  x  x , which yields the equation
Fig. 4.30 x v y v
As in Example 3 (Part n), to start with develop the
3v sin 
2
2v
equation that gives the moment of time t at which the t2  t  2  0.
velocity of the stone is perpendicular to its position g g
vector. From this equation find the values of  for If the velocity of the stone is never perpendicular to its
which it has no real solution. That’s it! position vector, the discriminant of the quadratic
We can develop the required equation in the following equation in t must be negative, that is,
2
ways.  3v sin    v2 
(i) Figure 4.31 shows the velocity and position vectors    8  2 .
  g  g 
of the stone at a certain instant t. If vt is perpendicular
   Thus, for the stone to be always moving away from the
to rt , we have rt  vt  0, which can be written as
thrower, we must have sin   8 / 9  0.94, i.e.,
v  < 70.5°. 
 
rt vt 4.4.2. Motion of a Body Thrown
 along the Horizontal
O x
Let us consider the motion of a body thrown along the
Fig. 4.31
horizontal and moving only under the action of the
  1 2  force of gravity. We shall again ignore the air
  2  
 
 (v cos )tiˆ   (v sin )t  gt  ˆj   v cos iˆ  (v sin   gt ) ˆj  0 resistance. Let us, for example, throw a particle from a
which gives tower, its initial velocity v0 being directed along the
horizontal, Fig. 4.33(a).
 3v sin   2v 2
t2    t  2  0. We shall analyze the motion of the particle onto the
 g  g (downward) vertical y- axis and the horizontal x- axis.
(ii) We can express the scalar product of the position The motion of the particle on the x- axis is the motion
and velocity vectors without resorting to components with zero acceleration at a velocity vx = v0. The motion
and proceed straightaway as below. in the y- direction is a free fall with an acceleration
  
We have rt  vt  0 v ay = g under the action of the force of gravity with zero
 initial velocity.
 1  2  
or  vt  2 gt   (v  gt )  0 v0
   O
x
g v0
or v 2 t  vgt 2 cos(90  )  1 vgt 2 cos(90  )  1 g 2 t 3  0
2 2 vx
which gives
 3v sin   2v 2
t2   t  2  0 . v
 g  g vy
y
(iii) The motion of the stone is given by the following (a) (b)
equations (Fig. 4.32):
Fig. 4.33
g
x  vt cos , y  vt sin   t 2 , The vx component remains constant and equal to v0. The
2
vx  v cos , v y  v sin   gt ,  v (say). vy component varies with time as: vy = gt. The resultant

81
 
velocity can be easily found with the help of the 1
parallelogram rule as shown in Fig. 4.33(b). 16 m  (16 m/s)t  10 m/s2 t 2
2
In the coordinate system shown in the figure on the which gives t  4 s. The above equation yields a
assumption that it was projected from the origin at t  0,
negative root also. Can any physical interpretation be
the coordinates of the particle at a moment t will be assigned to this root?
x = v0 t,
(b) How far from the base of the tower does the
1 2
and y gt . projectile hit the ground?
2 On multiplying vx and the time of flight we obtain the
In order to find the equation of the trajectory, we
x-coordinate of the point where the projectile falls on
express t in the first equation in terms of x and
substitute it into the second. This gives the ground: x  12 m/s  12 s  48 m.
g (c) What is the maximum height above ground attained
y  2 x2.
2v0 by the projectile?
Analyzing the y- component of motion we obtain
The graph of this function is shown in Fig. 4.33(b).
Such trajectories are called parabolas. Thus, a freely v02 y (16 m/s)2
hmax  h   16 m   28.8 m.
falling body with an initial horizontal velocity moves 2g 2  10 m/s 2
along a parabola.
The distance covered in the vertical direction does not (d) With what velocity the projectile lands on the
depend on the initial velocity. However, the distance ground?
covered in the horizontal direction is proportional to the The x- component of velocity remains constant and
initial velocity. Therefore, at a high initial horizontal equal to its initial value v0 cos   12 m/s, because the
velocity, the parabola along which the particle falls is
stretched in the horizontal direction. x- component of acceleration is absent. The
If we know the initial velocity v0 and the height h from y- component of velocity varies according to the
which the body is thrown, we can calculate the Formula 4.10. Substituting t  4 s into this formula we
horizontal range R to the place where the particle falls.
On substituting y = h and x = R, in the above equation obtain v y  16 m/s  10 m/s2  4 s  24 m/s. The
we obtain projectile lands on the ground with velocity
2h 
R  v0 . v  (12iˆ  24 ˆj ) m/s.
g
(e) The projectile just crosses a wall of height 8.8 m.
 A ball rolling off a table of 1 m height lands at a How sooner after the launch the projectile does this?
distance of 2 m from the edge of the table. What was When the projectile is just above the wall, its
the horizontal velocity of the ball? Neglect air displacement in y- direction is (16 m  8.8 m)
resistance. Take g  10 m/s 2 .  7.2 m . Using Formula 4.12 we obtain
 Example 7. Assume that a projectile is launched from 1
a tower of height 16 m with a velocity 20 m/s making an 7.2 m  (16 m/s)t   10 m/s 2  t 2 , which gives
2
angle of 53 with the horizontal, see Fig. 4.34. t  3.6 s. Why did we neglect the other root of this
y(ĵ) 20 m/s equation?
(f) With what velocity the projectile crosses the wall in
t=0 53
Part (e)?

v  vx iˆ  v y ˆj  v0 x iˆ  (v0 y  gt ) ˆj
Tower 16m
 12iˆ m/s  (16 m/s  10 m/s 2  3.6 s) ˆj
x(î)
Ground O  (12iˆ  20 ˆj ) m/s.
Fig. 4.34 (g) What is the horizontal distance between the wall
(a) How long does the projectile take to hit the ground? and tower in Part (e)?
Substituting v0 y  16 m/s , a y  10 m/s2 and xwall  v0 x  t  12 m/s  3.6 s  43.2 m.
S y  16 m, into Eq. 4.6b we obtain

82
(h) What is the equation of the path of the projectile in and 1 1
(150m/s)t  (10m/s2 )t 2  0  (t  T )  (10m/s2 )(t  T )2 .
the coordinate system given in the figure? 2 2
y First shot (fired at t = 0)
The x- and y- coordinates of the projectile vary with
time according to the laws (Fig. 4.35): 100 3 m/s Second shot (fired at t = T)
x  12t
60 150 m/s
y  16  16t  5t 2 . 60 1000 3 m/s
50 3 m/s
In order to find the equation of the trajectory, we
3000 m
x
substitute t  from the first equation into the P, t
12
second. This gives y
2 x
 x  x O
y  16  16    5   . x
 12   12 
Fig. 4.36
y
On solving these two equations we obtain T  20 s,
t (x, y) and t  40 s. This implies that the first shot travels for
t=0
time t  40 s before hitting the other shot.
16 m Displacement of the first shot in the x- and y- directions
in this time are
x
O S x  50 3 m/s  40s  2000 3 m  2 3 km
Fig. 4.35 and S y  (150 m/s)  (40 s)  1  (10 m/s) 2  (40 s)2
2
(i) At what moment of time the speed of projectile is  2000 m =  2 km.
24 m/s? Pay attention to the coordinate system given in the
Using v  vx2  v y2 we can write problem. The y- component of the displacement of shot
1 from the cannon is –2 km, that means 2 km
(12 m/s)2  ((16  10 t )m/s)2  24 m/s downward form the point of projection; the
y- coordinate of point P is 3 km  2 km  1 km.
which yields t  3.68 s.
The coordinate of point P are (2 3,1) km.
Example 8. A cannon situated on the top of a hill of
Alternatively, without delving on which shot was fired
height 3000 m fires two shots, each with the same
first, you can proceed by simply assuming their times of
speed 100 3 m/s at some interval of time, one shot motion until they collide. Assume the time of motion of
upwards at an angle of 60 with the horizontal and the the first shot is t1 and that of the second shot is t2 .
other shot horizontally. The shots collide in air at point
This immediately gives
P. Take g = 10 m/s2. Find
(a) the time interval between the firings; (50 3 m/s)t1  (100 3 m/s) t2
(b) the coordinates of the point P. Take origin of the 1 1
coordinate system at the foot of the hill right below the and (150 m/s)t1  (10 m/s 2 )t12  0  t2  (10 m/s 2 )t22
2 2
cannon. which give t1  40 s and t2  20 s. Now you can
You can put forward many arguments to show that the
proceed further.
shot fired at the angle of 60 to the horizontal takes a
longer time to reach point P (you must do it), therefore,  (i) What are the equations of the trajectories of the
it was fired first. two shots in the coordinate system given in the
Let the first shot be fired at t  0 , and the second shot, problem?
fired in the horizontal direction, at moment t  T , and (ii) Can a time interval between the firings ensure that
the two shots collide at point P at moment t  t. the two shots hit each other at the point which is at the
Figure 4.36 sketches the motion of the shots. On same horizontal level as the foot of hill? If yes, what is
equating the horizontal and vertical displacements of it equal to?
the two shots, since they reach point P simultaneously, Example 9. A man stands on the top a tower and
we get throws a ball at a speed of 10 m/s at an angle  to the
(50 3 m/s) t  (100 3 m/s)(t  T ) horizontal. The height of the tower is 10 m and the ball

83
strikes the ground at a distance of d from the foot of the solving. However, it is important to be familiar with
tower. Find the value of  for which the distance d is a such alternative methods. Let us project the ball with
maximum. Take g = 10 m/s2. speed 10 m/s at different angles from the horizontal,
Take a coordinate system whose origin coincides with and compute the horizontal distance of the point where
the base of the tower, x- axis is horizontal in the plane it lands on the ground from the base of the tower each
of the trajectory of the ball and y- axis is vertically time. The results of this experiment are complied in
upwards. (Fig. 4.37). Table 4.1
y v
Table 4.1: Dependence of d on 
, degrees t, s d, m
 0 1.41 14.1
15 1.70 16.4
10 m 30 202. 17.3
x 45 2.30 16.1
O
d 60 2.50 12.5
90 2.70 0
Fig. 4.37
Next we construct the d- graph, plotting  and d on
If the ball strikes the ground in time t, two mutually perpendicular axes, Fig. 4.38.
1
10 m  (10sin ) m/s  t   10 m/s 2  t 2
2 d, m
20
which gives t = sin  + 2  sin 2 .
15
Distance of the point where the ball strikes from the
base of the tower where is 10
d = (v cos)t
5
= (10 cos )  (sin + 2  sin 2  ).
On differentiation of the expression for d and some , degrees
rearrangement we obtain 0 15 30 45 60 90

dd
d

 10 sin   2  sin 2    sin   cos 2  
2  sin 2  
.
Fig. 4.38
From the graph it can be seen that as the angle of
dd projection  increases, the distance d of the point where
The condition  0 gives the value of  at which d the projectile falls from the base of the tower first
d
increases, attains a maximum value at a certain angle of
 projection and then decreases, becoming zero at
is maximum. For 0    , (sin   2  sin 2  )  0,
2   90. When  = 90, the projectile moves in vertical
therefore, we have
direction. It can be observed from the graph that d
cos 2  attains its maximum value d m  17.3 m at the angle of
 sin  +  0,
2  sin 2  projection   30.
which on simplification gives These solutions involve lengthy cumbersome calculations.
1   But one must be able to perform them when asked to. You
sin    0  must know the steps to be followed and the calculations
2  2
involved in graphical method. This will help you in your
or  = 30. practical classes. However, for such seemingly tedious
It is left to you to perform the second derivative test (or problems we have fortunately some short elegant methods
otherwise) to ascertain that at  = 30, the value of d is employing mathematics in ingenious ways. See the solution
maximum. We substitute   30 into the expression of this problem in Example 35. 
for d to obtain 4.4.3. Projectile Motion on an Inclined
 3  1 1
d m  10    2    10 3 m.

Plane
 2  2 4 A projectile is fired up an incline with an initial speed
For the beginners who are not introduced to calculus v0 at an angle  to the horizontal as shown in Fig. 4.39.
yet, here is a graphical method to arrive at the answer. The angle of the incline is  to the horizontal.
At this level we do not apply such methods for problem

84
 
We substitute  for  into the expression for R to
4 2
obtain its maximum value:
v02
v0 Rmax  .
R g (1  sin )


If the angle of projection is measured from the incline
plane, not from the horizontal, then x- and y-
Fig. 4.39 components of the initial velocity are v0 cos(  ) and
As in Section 4.4, we choose the coordinate axes so that v0 sin(  ) . Except this fact, every step in the above
the x- axis is horizontal and y- axis lies in the same vertical analysis remains in force, yielding a different
plane with the initial velocity v0 . The x- component of the expression for R , of course. Do you expect a different
initial velocity is v0 cos  while the y- component is expression for Rmax also?
v0 sin  (for the directions of x- and y- axes shown in Fig. Alternatively, we choose the coordinate axes so that x-
axis lies in the incline plane along the line of maximum
4.40). Since the x- component of acceleration is absent, the
slope, in the same vertical plane with the initial velocity
x- component of velocity remains constant and equal to the
v0 , and y- axis perpendicular to the chosen x- axis, see
initial velocity v0 cos  .
Fig. 4.41.
The y- component of motion occurs under the
acceleration due to gravity g . x

y ay = –g
P
y g sin  g cos 
v0
g
ax = 0
v0
v0 sin  v0 sin ( –   v0 cos ( – ) ax = –g sin 
R 
 ay = –g cos 

x
v0 cos  Fig. 4.41
Fig. 4.40 The x- component of initial velocity is v0 cos(  )
If the distance of the point P where the projectile falls and the y- component is v0 sin(  ) . The acceleration
on the incline from the point of projection is R, the x- of the projectile is g in vertically downward direction.
and y- coordinates of point P are R cos  and R sin  Resolution of g gives the components of the
respectively. If the projectile takes a time T to hit the acceleration ax   g sin , a y   g cos .
incline plane, the coordinates of point of impact are If the projectile falls on the incline at a point whose
obtained from equations:
coordinates are ( R,0), we have
x  R cos   (v0 cos )T
1
1 R  v0 cos(  )T  ( g sin )T 2
y  R sin   (v0 sin )T  gT 2 . 2
2
1
Substituting for T the from the first equation into the and 0  v0 sin(   )T  ( g cos )T 2 .
second and simplifying, we obtain 2
Substituting for T from the second equation into the
2v 2 sin(   ) cos 
R 0 . first we get, after some simple trigonometry,
g cos 2  2v 2 sin(  ) cos 
It can be seen that for a fixed value of v0 , the R 0 .
g cos2 
expression for R assumes its maximum value when the
This approach is distinctly advantageous as compared
function f  sin(  ) cos  is maximum. Using the to the previous one. Mathematical manipulations are a
first principal of maxima-minima or otherwise it can be great deal simpler. If you measure the angle of
shown that the function f has its maximum value projection from the inclined plane, instead of from the
  horizontal, you can further reduce the mathematics. We
when    . Consequently, the maximum value of present the bare essential steps leaving the required
4 2
manipulations to you
 
R is obtained when the angle of projection    .
4 2

85
1  2v 2 
0  (v0 sin )T  ( g cos )T 2 d    tan  sec .
2
 g 
1
R  (v0 cos )T  ( g sin )T 2 . (ii) The time taken by the projectile to hit the plane
2 d cos 
Substituting for T from the first equation into the t
v
second we get
2v 2 sin  cos(  )  2v 2 tan  sec  
R 0 .  g  cos   2v 
g cos2        tan .
df v  g 
Now assume sin  cos(  )  f and set  0. You We can also find t by dividing the second equation
d
  developed in Part(i) by the first
get    . Hence 1 2
4 2 gt
d sin  2
v02 
Rmax  . d cos  vt
g (1  sin  )
gt
 Demonstrate that if the projectile is fired with initial or tan  
2v
velocity v0 down the incline, then the maximum range is
 2v 
or t    tan .
v02  g 
Rmax  .
g (1  sin ) (iii) The y- component of the velocity of the projectile
 Example 10. A particle is thrown horizontally with a at the moment it hits the plane is
speed v from a point on a plane inclined at an angle  to  2v 
vy = 0 + g t = g    tan  = 2v tan .
the horizontal. The trajectory of the particle lies in the  g 
vertical plane that contains the line of maximum slope Required velocity
on the incline. (i) How far from the point of projection
will the particle land on the plane? (ii) How long does v  vx2  v 2y  v 2  (2v tan )2  v 1  4 tan 2  .
the particle take to do it? (iii) What is the velocity of the The velocity makes an angle of
particle at the moment it hits the plane? vy
The choice of coordinate system in this problem is   tan 1  tan 1 (2 tan  ) from the horizontal.
vx
more than obvious. Choose the x- axis in the horizontal
direction and y- axis in the vertically downward Example 11. A small, elastic ball is dropped vertically
direction, (Fig. 4.42). onto a long plane inclined at an angle  to the
The components of acceleration along the chosen horizontal. Is it true that the distances between
coordinate axes are a x = 0, ay = g. Also, consecutive bouncing points grow as in an arithmetical
at t = 0, x = 0, y = 0, u x = v, vv = 0, progression? Assume that collisions are perfectly elastic
and at t = t, x = d cos , y = d sin . and that air resistance can be neglected.
(i) For the motion in x- and y- directions we obtain You will learn about collisions in Chapter 9. Here, it suffices to know
d cos  = v t that on impact, the component of velocity of the ball that is
perpendicular to the plane gets reversed, and the component along the
1
and d sin  = 0  t + g t 2 . plane remains unchanged.
2 We know how to calculate the displacement undergone
v
in the nth second in rectilinear motion with constant
O x acceleration. This problem is no different. After a little
rearrangement the problem can be reduced to a simpler
problem as depicted in Fig. 4.43.
d
It can be immediately seen that the time interval
y P between any two successive impacts has be to the same,
 2u
T . The distances between the consecutive
Fig. 4.42 g cos 
Substituting for t from the first equation into the second bouncing points can be easily calculated as under:
and solving for d we obtain 1
S1  uT  g sin (T 2 )
2

86
1  1  The difference S   S is equal to the area of the
S 2  u (2T )  g sin  (2T ) 2   uT  sin T 2 
2  2  rectangle ABCD . This is all that you need to conclude
that the distances between the consecutive bounces in
1
 uT  g sin  (3T 2 ) the problem are in arithmetical progression.
2 Further, you can also argue intuitively that the motion
1  1  in the direction perpendicular to the plane consists of
S3  u (3T )  g sin  (3T ) 2   u (2T )  g sin  (2T ) 2 
2  2  bounces of identical heights, i.e., of identical periods.
1 The component of the take off velocity perpendicular to
 uT  g sin  (5T 2 )
2 the plane is same after each bounce, and also the
1  1  acceleration component perpendicular to the plane is
S 4  u (4T )  g sin  (4T ) 2   u (3T )  g sin (3T ) 2  same g cos  throughout the motion. And, since the
2  2 
1 acceleration along the plane is constant, the ball's
 uT  g sin  (7T 2 ). average speed between bounces increases uniformly,
2 and so the distances between two consecutive bounces
It is obvious that S1 , S 2 , S3 ,... form an arithmetical increase in an arithmetical progression. 
progression.
u 4.4.4. Flight of Bullets
When the velocities of projectiles are high, air resistance considerably
g cos  alters their motion in comparison with the results of calculations
g sin 
u carried out in the previous sections. If air resistance were absent, the
maximum range of the projectile would be observed, as was
mentioned in Section 4.4.1, at the angle of projection equal to 45°.
The effect of air resistance on the flight of projectiles becomes
weaker for larger projectiles for the reason the mass of a projectile
increases as the cube of its size, while the force of air resistance
increases as the square of its size. Thus, the ratio of air resistance to
the mass of the projectile, i.e., the effect of air resistance, decreases
with the increasing size. Therefore, for the same initial velocity of
 projectiles fired from a gun, their range increases with the calibre.
It can be shown that air resistance leads to a change in the trajectory
of a bullet such that the angle of inclination corresponding to the
maximum range turns out to be less than 45° (it is different for
different initial velocities of the bullet). At the same time, the
g sin  horizontal range (as well as the maximum height of the flight) turns
out to be much smaller. For example, for an initial velocity of
g cos  800 m/s and angle of 45°, the horizontal range of the bullet is
u (800 m/s) 2 sin 90
u  64000 m =64 km in the absence of the
10 m/s 2
S1 S2 S3 resistance of the medium and assuming that acceleration due to
gravity in constant 10 m/s2 at each point of the trajectory. However,
Fig. 4.43 for the same initial velocity, the maximum range of flight does not
You can also compute the displacements in time nT exceed 3.2 km, i.e., is reduced to less than 1/20 of the theoretical
value. The maximum height attained by the bullet is reduced almost
and in time (n – 1)T. Then compute S( nT )th , and show in the same proportion.
The most advantageous angle of firing approaches 45°. Long-range
that S’s are in arithmetical progression. guns fire at an angle close to 45°. Since projectiles rise in this case to
Alternatively, Fig. 4.44 shows the velocity-time graph of a larger height, where the density of the atmosphere is lower, the
a motion with constant acceleration. Let the displacement effect of air resistance becomes less noticeable.
undergone in the time interval (4T  3T ) be denoted by
S and that in the interval (5T  4T ) by S  . 2
1
v
A 45°
D C C B
B
A Fig. 4.45

S S If the target C is at a distance less than the maximum range AB


(Fig. 4.45), the projectile can hit the target in two ways: at an angle of
inclination which is either less than 45° (grazing firing) or larger than
t
O T 2T 3T 4T 5T 45 (steep firing).

Fig. 4.44

87
4.5. Uniform Circular Motion Suppose that in a small time interval t the position
vector of the point rotates through the angle  and
Consider a point that moves along an arbitrary x- axis at    
change in its position is r  r2  r1 . Since v is always
velocity v1 . Subsequently the point turns at right angles 
perpendicular to r , these two vectors change their
and moves along y- axis at velocity v2 . The velocity of
directions by the same angle in any time interval. In the
the point changed in direction and if v2  v1 , it changed   
vector diagram for the equation v2  v1  v , we know
in magnitude as well. In case v2  v1 , the change in   
that | v2 |  | v1 | . The direction of v is perpendicular
velocity arises solely because of change in the direction 
to r directed toward the centre O of circle, radially
of motion. What is the direction of average acceleration
   inward along the bisector of the angle  drawn within
of the point? Figure 4.46 shows that v  v2  v1 is the circle. The triangles OAB and APQ are similar
directed inside of the curve. Hence, the average isosceles triangles. (How?)

 v It follows from the similarity of the triangles that the
acceleration aav  is also directed inside the curve.
t ratios of their analogous sides are equal:
 
 | r | | v |
v2   ,
v1 r v
y
   v 
v v2 which gives | v |    | r | .
 r

x
v1 Since, t is very small, | r |  vt , and therefore we

| v | v 2
Fig. 4.46 finally get  .
t r
If the point navigates a turn broken in two stages 
 | v | 
(Fig. 4.47(a)), it is subjected to two accelerations. If the From the definition a  lim   , we find that the
turn is broken in three stages, the point will have three  t  0  t 

accelerations, as shown in (b). What about the magnitude of centripetal acceleration is


acceleration of the point when the number of stages v2
increases? The acceleration appears as in (c) as the ar  . …(4.17)
r
number of stages is increased.
The subscript r in ar (at times denoted by aN also)
When the line segments merge into an arc of a circle,
the instantaneous acceleration is directed radially indicates that the acceleration is in radial direction. As a
inward, toward the centre. This acceleration is called vector equation we would write
the centripetal (centre-seeking or directed toward  v2
centre) acceleration. ar   rˆ
r
where r̂ is the radial unit vector directed outward as
a a shown in Fig. 4.49. The figure also shows the velocity
a aa and acceleration of the point at three arbitrary points on
the circular path.
(a) (b) (c) 
v2

Fig. 4.47 
r̂3  a2
r̂2
a3
Let us proceed to analyze the circular motion
quantitatively. Figure 4.48 shows a point moving at 
v3 C
constant speed v in a circle of radius r.
 
v2 a1 
v1

r̂1
 B v1   
r2
 Q v P v1  v2  v3  ...
r   
O  | v1 |  | v2 |  | v3 |  ...  v
     
r
1
v2 v1 a1  a2  a3  ...
A 
   v2
A | a1 |  | a2 |  | a3 |  ... 
r
Fig. 4.48 Fig. 4.49

88
Can you write the expression for centripetal directed along the tangent to the circle as shown in the
acceleration of the point in terms of time period? The figure.
period T is the time it takes to complete one revolution. Another derivation that employs polar coordinates is
The point travels a distance of 2r in one revolution, given in Appendix 1. Students are advised to learn these
so the speed is v  (2r ) / T . derivations, as they turn out to be a formidable tool for
As an alternative, the above derivation can be done in problem solving.
an absolutely simple way using calculus.  Example 12. The coordinates of a particle at time t
Consider a point moving on a circle with constant are given by x  3sin 5t and y  3cos 5t . What is the
v speed of the particle?
angular velocity   , where v is the speed of the
r To get vx and v y we differentiate the corresponding x-
point and r is the radius of the circle, Fig. 4.50.
and y- coordinates
y ( ˆj ) dx dy
vx   15cos 5t , v y   15sin 5t .
P dt dt
r sin t  Hence the speed of the particle
r
t v  vx2  v y2  152 cos 2 5t  152 sin 2 5t  15 units.
x (iˆ)
O r cos t
Show that the trajectory of the particle is a circle. 

4.5.1. Acceleration in Curvilinear


Fig. 4.50
Motion
Consider a point moving along a curved path, as shown
If the point is on positive x- axis at time t  0,
 in Fig. 4.52. In general, both the magnitude and the
POx  t. The position vector r of the point can be direction of the velocity may vary along its path. The
written in terms of unit vectors iˆ and ĵ as radial acceleration associated with changes in the
 v2
r  r cos t iˆ  r sin t ˆj. direction of the velocity is ar  , directed toward the
 r
On differentiating the position vector r relative to
time twice we get the acceleration vector. Note that r centre of curvature, where r is the radius of curvature of
and  are constant. the path at the given point. A small segment of the path
 can be treated as an arc of a circle. The figure shows
 dr
v  r sin t iˆ  r cos t ˆj one such circle. The radius of the circle thus generated
dt is the radius of curvature of the path at that point and its

 dv centre is the centre of curvature.
and a   2 r cos t iˆ  2 r sin t ˆj.
dt
 a
By looking at the components of a we immediately
2
v v2
conclude that its magnitude is 2 r    r  and it
r r ar
 
must be directed opposite to r . Vector r is directed

from centre O to the point P, therefore, a must be
directed from point P to the centre O. See Fig. 4.51.
y ( ˆj ) Fig. 4.52
r
When the speed of the point varies with time, there is
2 r cos t P
also an acceleration along the tangent to the path:
2r 2 r sin t dv
a  , …(4.18)
t x (iˆ) dt
O in the direction of the velocity if the speed is increasing,
and opposite to velocity if the speed is decreasing. The
resultant acceleration of the point is the vector sum of
these two components:
Fig. 4.51   
 a  ar  a .
From the expression for the velocity vector v you can  
Since ar and a are always perpendicular, the
conclude that the magnitude of velocity is r and it is
magnitude of the resultant acceleration is

89
where C is a constant of integration.
a  ar2  a2 .
At t  0, v  0 , hence C  0.
In the special case of motion of a point in a circle, it is
Thus the speed of the particle as a function of time is
sometimes convenient to use the unit vectors ̂ and r̂
t2
shown in Fig. 4.53, where r̂ is directed radially v .
2
outward from the centre and ̂ is in the direction of The radial acceleration of the particle is
increasing . The magnitudes of these unit vectors are v2 t 4
constant (equal to unity), but their directions change in ar   .
time. The acceleration of the point is expressed as r 8
The angle (which is given to be 45 at instant t)
 v2 dv
a   rˆ  ˆ . …(4.19) between total acceleration and velocity is same as the
r dt angle between total acceleration and tangential
y acceleration, see Fig. 4.55. (Is this always true? What if
̂ r̂ the particle were to decelerate?)

 x

t4
ar 
8 45
t2
Fig. 4.53 v
a  t 2
In uniform circular motion dv/dt = 0, so the acceleration
has only the radial term. Fig. 4.55

 Consider a particle moving in a circle with a variable From the figure we have
speed, which means the angular velocity  varies with t4
 t3
time. From the expression r  r cos t iˆ  r sin t ˆj tan 45  8 
derive Eq. 4.19. t 8
which gives t  2 s. 
 Example 13. The tangential acceleration a of a
particle moving in a circle of radius 2 m varies with 4.6. Relative Velocity
time t as in Fig. 4.54. Initial velocity of particle is zero. We shall now study the cases when one of the reference
Find the time after which the total acceleration of the frames is in motion relative to another frame. Clearly,
particle makes an angle of 45 with velocity? the second reference frame is also in motion relative to
a (m/s2) the first one.
To begin with students are advised to read the following
questions, and try to answer them intuitively or
otherwise, before learning the concept and formulations
of the relative velocity formally. You may not be able to
45 answer them correctly, you must try nevertheless.
t (s) (i) A girl is in a rectilinear non-uniform motion on a
platform and the platform moves uniformly in a straight
Fig. 4.54 line relative to the ground, can the motion of the girl as
From the graph we can express the tangential seen from the ground be along a curved path?
acceleration a as a function of time t . We have (ii) Under what condition is the motion of the girl
rectilinear in (i), even if she is in non - uniform
a  (tan 45 )t rectilinear motion on the platform?
or a  t (iii) A boat covers a certain distance downstream in less
dv time than it takes to cover the same distance
or  t. upstream. Why?
dt
(iv) A swimmer covers 3 km in 3 hours in still water,
On integration we get and a log covers 1 km downstream during the same
t2 time. What distance will be covered by the swimmer
v   C,
2 upstream in the same time?

90
(v) A crawler tractor moves at a velocity of 5 m/s. What 1s 2s 3s
is the velocity of the (a) upper, and (b) lower parts of rBA
A 1m 2m 3m
the crawler relative to the ground? What are the
(a)
velocities of these parts relative to the tractor?
The motion of any body has to be described relative to
rBA, m
some frame of reference, such as the ground.
Sometimes it is necessary to examine the motion of one 3
body relative to another body that is also moving
relative to the ground. For one-dimensional motion, it is 2
easy to determine the velocity of one body relative to
another. As an example, consider the rectilinear motion 1
of two points.
Assume that two points A and B move on parallel t, s
straight lines with velocities 1 m/s and 2 m/s 0 1 2 3
respectively relative to the ground. Let the points be (b)
side by side at the initial moment t = 0, Fig. 4.56.
Fig. 4.58
B 2 m/s
Therefore, in this example you can obtain the velocity
of point B relative to point A by subtracting the velocity
A 1 m/s
of A relative to ground from the velocity of point B
Fig. 4.56 relative to ground.
vBA = vBG – vAG.
Positions of the points A and B at time t = 0, 1 s, 2 s, 3 s
Proceeding in the similar way, we can obtain the
are drawn in Fig. 4.57.
position vector of A relative B at moments t = 0, 1 s,
2m 2m 2m 2 s, 3 s…, (Fig. 4.59(a)), and can generate the position
B 2 m/s
versus time graph for the motion of point A as seen
t=1s t=2s t=3s
from the reference frame fixed at point B as in (b).
1 m/s
Computation using the graph of Fig. 4.59(b) shows that
A t=1s t=2s t=3s the velocity of A relative to B is –1 m/s. Also,
1m 1m 1m vAB – vBG = 1 m/s – 2 m/s = –1 m/s. Hence, we
conclude that
Fig. 4.57 vAB = vAG – vBG.
Now, imagine an observer fixed on point A who looks t=3s t=2s t=1s B
at point B. Equivalently, plant yourself at A and look at rAB
B. What do you observe? At initial moment t = 0, point –3 m –2 m –1 m
B is on your side, 1 second later B is 1 m ahead of you, (a)
2 seconds later B is 2 m ahead of you, 3 seconds later B rAB, m
is 3 m ahead of you, and so on. We shall represent the
position B as observed by you (which is also the 0
1 2 3
t, s
position of B relative to A) by the symbol rBA , and in
–1
the same notation velocity of B relative to A as vBA .
The position of point B relative to point A is shown in –2

Fig. 4.58(a) and plotted in (b).


We shall apply Eq. 4.1 to the history of motion of point –3

B as ‘seen’ from point A to compute velocity of B


(b)
relative to A as
1 m0 2 m0 3 m0 Fig. 4.59
vBA     1m/ s.
1 s 0 2 s0 3s  0 Now we shall consider the case in which the points A
If you subtract the velocity of A relative to the ground and B move in a plane in different directions. As a
from the velocity of B relative to the ground, you get specific example, let us assume that A and B move in
1 m/s: mutually perpendicular directions with constant
vBG – vAG = 2 m/s – 1 m/s = 1 m/s. velocities 1 m/s and 2 m/s as shown in Fig. 4.60(a). We
This is exactly the velocity of B as calculated by the also assume that the positions of A and B coincide at the
observer on A from the position versus time graph. initial moment t = 0. The positions of points A and B are
drawn at the moments t = 0, 1 s, 2 s, 3 s in figure (b).

91
  
t=3s B or rAB  rAG  rBG .
y A

2 m/s t=2s
 
rAG rAB
3 5m
t=1s B

2 5m
B 1 m/s
5m rBG
A A
G x
t=0 t=1s t=2s t=3s
Fig. 4.62
(a) (b)
On differentiating both sides of the above equation
Fig. 4.60 relative to time we get the relationship between the
The position vectors of point B as seen from A at the velocities -
  
moments t = 0, 1 s, 2 s, 3 s are drawn in Fig. 4.61(a). v AB  v AG  v BG . …(4.20)
t=3s Let us apply the concept of relative motion to a few
3 5m simple problems.

t=2s
 Example 14. Let us solve Example 8 of Chapter 3 in
 a reference frame fixed at point A.
2 5m v AG  1 m/ s
 Calculations in the reference frame fixed at the point A:
  vBG  2 m/ s Initial velocity of B, uBA =  4 m/s.
t=1s vBG  v AG
Initial position of B, rBA = + 6 m.
5m  5 m/ s
Acceleration of B, aBA = + 1.2 m/s2
 tan  = 2
tan  = 2  At the moment points B hits A, rBA become zero.
A Hence, displacement of B is S BA  6 m.
(a) (b)
1 2
Fig. 4.61 Using equation S = ut + at we get
2
Now, the velocity of point B relative to A can be
1
obtained by dividing the changes in position vector by 6 m = (4 m/s)t + (+1.2 m/s2) t2
the corresponding time intervals. That is, 2
 5 m0 2 5 m0 3 5 m 0
which gives t = 2.3 s, 4.4 s.
vBA     5 m/ s,
1s0 2 s0 3 s0 t=0
in the direction of change in position vector, at an angle A 4 m/s B
 where tan  = 2, as shown in the figure. 6m
We can show that we get this same velocity by
subtracting the velocity of A relative to the ground from Fig. 4.63
the velocity of B relative to the ground, Fig. 4.61(b). Let us visualize the motion of point B when it is
Arguing in the same fashion we can show that on observed from point A. Initially point B was located
subtracting the velocity of B relative to the ground from 6 m away from the A and had a velocity of 4 m/s
the velocity of A relative to the ground we get the directed toward A (Fig. 4.63). It moves towards the A
velocity of A relative to B with a reducing speed as the acceleration is in the
   opposite direction. (Initial velocity is directed towards
v AG  vBG  v AB .
left and acceleration towards right.) Point B crosses
In the above examples the points were in uniform
point A at moment t  2.3s, goes backwards comes to
motion. Now we shall show that the above formula is
valid for non-uniform motions as well if we assume that rest momentarily, starts moving in the forward
   direction, crosses A again at t  4 s and then
v AB , v AG and vBG are the instantaneous velocities
taken at the same instant of time. Let the position surges ahead.
vectors of the point A and B from G (an origin on the Example 15. Two particles 1 and 2 have position
    
ground) be denoted by rAG and rBG . See Fig. 4.62. vectors r1 and r2 at time t, where r1  (1  t 2 )iˆ  ˆj and

Let us denote the position vector of A from B by rAB . 
r2  3iˆ  t 2 ˆj where r is in meters and t is in seconds.
These position vectors are related as
   Both the particles start moving when t = 0. At what
rBG  rAB  rAG
moment of time they are closest together.

92
The position vector of particle 2 relative to particle 1 is and in the time interval 8 s  t  12 s,
  
r21  r2  r1  (2  t 2 )iˆ  (t 2  1) ˆj. y21  (40t  5t 2 )  ( 240)  240  40t  5t 2 .
The distance d between the particle 1 and 2 is given by Table 4.2 depicts the essentials for generating the
d 2  (2  t 2 ) 2  ( t 2  1) 2 y21 -t graph, which is sketched in Fig. 4.65.
 2t 4  2t 2  5. Table 4.2: Relative position of the second stone as seen
from the first.
dd 2
d 2 has stationary values when  0,
dt (a) 0  t  8s
1 t, s 0 2 4 6 8
or 8t 3  4t  0  t  0,  . y21 , m 0 60 120 180 240
2
d2
Second stone
For 0  t  8 s , y21  30t
y21
For 8s  t  12 s, y21  240  40t  5t 2
t First stone
Fig. 4.64 (b) 8s  t  12s
The graph of d versus t (Fig. 4.64) shows that d 2 is
2
t, s 8 9 10 11 12
1 y21 , m 240 195 140 75 0
minimum when t   . Therefore, the particle 1 and
2 It is instructive to draw the variation of relative position
1 of the second stone from the first qualitatively. An
2 come to closest together at t  s.
2 intuitive feeling of the pattern of variation of the
 What is the least separation between the particles? relative position with time is as important as the step-
by-step solution.
Example 16. Two stones are thrown up simultaneously We shall now proceed to apply the concept of relative
from the edge of a cliff 240 m high with initial speed of velocity to some common problems, particularly the
10 m/s and 40 m/s, respectively. We shall draw the rain-umbrella problem and the problem of crossing a
graph of the time variation of position of the second river.
stone with respect to the first. (Assume stones do not
rebound after hitting the ground. We will neglect air y12, s
240
resistance, take g = 10 m/s2 for the sake of 220
calculations). 200
One can easily conclude that the stone thrown up with 180
velocity 10 m/s will reach ground first. This is so 160
because as long as both the stones are in motion the 140
other stone will always have a velocity of 30 m/s in 120
upward direction relative to it. Further, if the stones 100
take time, t1 and t2 to reach the ground, using Eq. 3.12, 80
60
1
we have 240  10t1   10t12 , which gives t1  8 s 40
2 20
1 t, s
and 240  40t2  10t 22 , which gives t2  12 s as 0 2 4 6 8 9 101112
2
their times of motion. At t  8 s , the first stone hits the Fig. 4.65
ground and comes to rest; the second stone continues
Example 17. In the absence of winds, raindrops fall
moving for another 4 seconds. This demands us to 
calculate the relative position of second stone from the vertically downward, say with velocity vro . A girl
first in two time intervals, 0  t  8 s and standing on the ground has to hold her umbrella in
8 s  t  12 s. vertical position to protect herself from the rain,
Fig. 4.66(a). If the girl starts walking to the right with
In time interval 0  t  8 s, measuring the height from 
velocity vgo relative to ground, in which direction she
the point of projection,
y21  (40t  5t 2 )  (10t  5t 2 )  30t , must hold her umbrella?

93
The girl will instinctively turn her umbrella and hold it velocity of the raindrop relative to ground as vro sin 30.
at a certain angle from the vertical so as not to get
This component must be equal to the velocity with
drenched. We can calculate precisely the direction in
which the girl runs. Therefore,
which she will hold her umbrella. Formula 4.20 can be

used the relate the required velocities. Velocity vrg of vro sin 30  4 m/s
the raindrops relative to the girl is given by the equation
   giving
vrg  vro  vgo .
vro  8 m/s.
As shown in Fig. 4.66(b), the raindrops appear to her
falling at an angle with the vertical. We have Also, velocity of raindrops relative to girl
vgo
tan   . vrg  vro cos 30
vro
While walking, the girl will be holding her umbrella at 3
 (8 m/s)   4 3 m/s.
angle  with the vertical as shown in figure (c). 2
As an effort to enhance confidence you can apply
Formula 4.20 in a formal way. The velocity of
 

 raindrops with respect to ground (vro ), the velocity of
vrg

 girl relative to ground (vgo ) and velocity of raindrops

vro 
vgo with respect to girl (vrg ) are related as
  
(a) (b) (c) vrg  vro  vgo .
Fig. 4.66
In our example, velocity of the girl relative to ground
Example 18. A girl standing on ground has to hold her must be such that when her velocity is subtracted from
umbrella at 30° with the vertical in order to protect 
vro , we get a vector directed vertically downwards.
herself from the rain, see Fig. 4.67(a). She throws the
umbrella and starts running at 4 m/s and finds raindrops Figure 4.69 illustrates the required vector operation.
falling vertically downward (figure b). In which
vro sin 30
direction the girl is running? Find the velocity of
raindrops relative to (a) the ground, (b) the running girl.
30 
vrg 30
30
90 
vro
vt0 
vro cos 30 vgo

Fig. 4.68 Fig. 4.69


(a) (b) It is left to you to calculate vro and vrg from the
Fig. 4.67 triangle of velocities.
Suppose you are running on a straight road besides a cart Example 19. A boy running in a straight line on a plain
with a speed that is equal to the speed of the cart itself. horizontal ground at 4 m/s finds that the rain drops fall
Will you surge ahead of the cart or be left behind? at angle 30° with the vertical. He reduces his speed to
Answer this question and apply the same line of 2 m/s and finds the rain falling vertically. Find the
reasoning to this problem. The girl will find the raindrops speed of the rain and the angle its velocity vector makes
falling vertically downward if she runs in the direction of with the vertical.
and with the velocity of the horizontal components of the With the assumed velocity of the rain (Fig. 4.70(a)) and
velocity of raindrops relative to the ground. given velocities of the boy (b), we construct the
Relative to the ground the raindrops are falling at an velocity diagram (c). That’s it! This problem involves
angle of 30 with the vertical, say, with speed vro . just this much physics. Now find vrg and  (in
From the velocity diagram of Fig. 4.68, you 5-6 seconds).
immediately get the horizontal component of the

94
downstream by the flowing water. Indeed, the upstream
 component of the velocity v1 of boat relative to water
 vrg

4 m/s (Initially)
must be equal to the river flow velocity v2 . See the
 2 m/s (After reduction) velocity diagram of Fig. 4.71(c). The angle  in the
 
figure must be such that the sum of vectors v1 and v2 is
(a) (b)
A directed along line AB. What is the velocity vbg with
which the boat crosses the river in this case? From the
30 

vrg
velocity diagram vbg  v1 cos  which is equal to

v12  v22 (using Pythagoras). How long does the boat


C
D 
B take to cross the river?
–4 m/s vbg –2 m/s
b
(c) t , obviously.
v1  v22
2

Fig. 4.70 Can the boat cross the river in lesser time (the point
where it lands on the other bank is not important)? We
Example 20. A boat can be sailed at speed v1 relative can use the velocity diagram (c) to answer this question.
to still water. The sailor wants to cross a river of width If the boat is headed as shown in the figure, it crosses the
b flowing at speed v2 . (i) In which direction should the river with velocity v1 cos .
boat be headed to get straight across? How long does it Apparently, the sailor has no control over v1 (we are
take in crossing the river in this case? (ii) If the boat is analyzing the problem under the assumption that the boat
pointed straight across, how long does the crossing is sailed on water with a constant speed) but he can
take?
choose  as he desires. In that case, to minimize the time
From formula 4.20, we see that the velocity of boat
 of crossing the river, he will try to make v1 cos  as large
relative to ground vbg is related to the velocity of boat
 as possible. The maximum value of the function cos 
relative to water v1 and the velocity of water relative to occurs at   0; therefore, the time of crossing the river

ground v2 , which the river flow velocity, as is minimum when the boat is pointed straight across, see
  
vbg  v1  v2 . Fig. 4.72.
B C
As a first step to visualize the motion of the boat consider
two simple cases. If the boat is headed downstream, it
will move with velocity vbg  v1  v2 in downstream
direction, Fig. 4.71(a), as seen from the bank. And if the v2
boat is headed upstream it will move with velocity v1 v12  v22
vbg  v1  v2 in upstream direction as in (b).
A
B
Fig. 4.72
v2 Under this condition the boat lands on the other bank at
(River) b a point somewhat downstream (at point C in the
vbg = v1 + v2 vbg = v1 – v2 AC
v2
figure), and takes a time in crossing the
v1 v2 v1 vbg v12  v22
v1 
v2 river. A useful result to note is that this time is also
A b BC
equal to or .
(a) (b) (c) v1 v2
Fig. 4.71 Example 21. A swimmer starts from point A on one
Now, suppose the boat is at point A on one bank and bank of a river and wants to reach point B, swimming
the sailor wants to sail it to point B right across on the directly along line AB, see Fig. 4.73. Swimmer’s speed
other bank taking the boat straight across. The sailor with respect to water is 2 km/h, which is also the speed
must head the boat somewhat upstream so as to with which the river flows. Find the angle at which the
compensate for the distance the boat is carried swimmer must aim.

95
B 2
2
River l12  l22 l12  l22
2km/h v2

l2 l2
60 tan  
l1
 
A O l1 1 1
v1
(a) (b)
Fig. 4.73
Fig. 4.75
Let the swimmer aim at angle  (  60 ) as shown in
For obtaining the velocity of particle 1 relative to the
Fig. 4.74. The angle  must be such that the velocity of chosen reference frame subtract the velocity of particle
the swimmer relative to bank is directed along line AB. 2 from the velocity of particle 1, see Fig. 4.76(a).
B v12

P
  
vr
 v12  v1  v2
 v2 2 
2 km/h
 
2 cos 

30 v1
l12  l22
A vrg 
v12  v12  v22 ,
(2 + 2 sin ) l2
v2 tan   v12  v12  v22
tan   l1 
Fig. 4.74 v1
 v2
tan  
When the velocity of river relative to ground vrg is 1 v1
(a) (b)
added to the velocity of the swimmer relative to water
vsw one obtains the velocity of the swimmer relative to Fig. 4.76
It can be seen that particle 1 is closest to particle 2 at
ground vsg . The addition of the velocities is also 
the instant v12 becomes perpendicular to the line
depicted in the figure.
From the figure we have joining them. How? In Fig. 4.76(b), the least distance
between particles 1 and 2 is the length of the line 2P.
2  2sin 
tan 60  From the figure,
2 cos  v2 l2

1  sin  tan   tan  v1 l1
or 3 | tan(  ) |  
cos  1  tan  tan  v l
1 2  2
or 3 cos   sin   1 v1 l1
3 1 1 | v2 l1  v1l2 |
 cos   sin   
.
2 2 2 v1l1  v2l2
 sin 60 cos   cos 60 sin   sin 30 The minimum distance between the particles 1 and 2 is
or sin(60  )  sin 0 d min  l12  l22 | sin(  ) |
or 60    30 | v2 l1  v1l2 |
= l12  l22
or 
  30 . (v2 l1  v1l2 )2  (v1l1  v2 l2 ) 2
Example 22. Let us consider the problem of | v 2 l1  v1l 2 |
= .
Example 30, Chapter 3. v12  v 22
We shall solve the problem in a reference frame fixed We can also calculate the instant of time at which the
to particle 2 and moving with it. distance between the particles is the least. To reach
At the initial moment t  0 the separation of particle 1 point P the particle 1 has to cover a distance
from particle 2 is l12  l 22 as shown in Fig. 4.75. l12  l22 cos(  ) with speed v12  v22 . Therefore
particle 1 takes time

96
l12  l22  | cos (  ) | d min  (1km)sin 60 
3
km.
t 2
v12  v22 Before stepping into the shoes of person 1, you
l12  l22 v1l1  v2 l2 v1l1  v2l2 calculated the velocity of the person 2 relative to person
   1. Do it again. You determined certain angles and
v12  v22 (v2l1  v1l2 ) 2  (v1l1  v2l2 ) 2 v12  v22
distance. Determine them again.
to reach point P. Since the initial moment of time is t  0,
Example 24. A balloon carrying a man ascends at the
the instant of time at which the distance between the two rate of 3 m/s (assumed constant). At t = 3 s (assuming
particles is the least is given by the above expression. t = 0 to be the instant when the balloon is at the ground
This approach is best suited when we are given the level), the man drops a ball (I) and simultaneously
numerical values of the initial distances and velocities. throws another ball (II) with a velocity of 1 m/s in the
In that case, calculations of the distance and angles is a
horizontal direction. Take g  10 m/s 2 . Find the
game. As an example, take l1  10 3 m, l2  10 m,
distance between the two balls at t = 4 s.
v1  2 m/s and v2  2 3 m/s, and solve the problem. At t = 3 s, height of the balloon from the ground level
Also see Example 23. = 3 m/s  3 s = 9 m.
Velocity of the ball (I) when it is dropped = 3 m/s
Example 23. Two persons, 1 and 2, are walking with
upwards. Ball (II) thrown with a velocity of 1 m/s in the
constant velocities of equal magnitude along two roads horizontal direction also has a velocity of 3 m/s in
which are angled at 60 to each other as shown in Fig. 4.77. upward direction.
Displacement of ball (I) dropped from the balloon in in
2
one second time (from t = 3 s to t = 4 s) is
3 km
1
S = 3 m/s  1 s + (10 m/s 2 ) (1 s) 2  2 m, in
2
60 vertical direction.
Horizontal component of the displacement of ball (II) in
4 km one second time is 1 m/s  1 s = 1 m. Vertical
1
component of the displacement of the ball (II) in one
second time is –2 m. (How?) In vertical direction both
Fig. 4.77 the balls undergo same displacement. Therefore, the
separation between the balls at t = 4 s is 1 m.
Find the shortest distance between the persons during
Alternatively, velocity as well as acceleration in the
the course of their motion?
vertical direction of ball (II) relative to ball (I) is zero.
The problem was thrown to you by a friend standing
The balls will always lie in same horizontal plane.
somewhere on the ground, may be at the intersection of
Moreover, ball (II) does not have any acceleration
the roads. He expected you to use geometry to set
relative to ball (I) in horizontal direction. Separation
algebra and then apply the concepts of maxima and
between the balls at t  4 s is, therefore, 1 m. 
minima of calculus and find the answer after some
irritating mathematics. But you are a little smarter, not
interested in doing lengthy calculations, though you can
4.7. Velocity of approach
do it when asked to. All you did is this: you stepped In Section 3.2, Chapter 3, we introduced two important
into the shoes of person 1 and captured his sight. quantities, namely distance covered by a moving point
Whatever you saw you sketched that diagrammatically, and distance between two points. Distance between two
Fig. 4.78, and came with the answer, in 10 seconds. points or separation between two points is defined as
the length of the straight line segment joining the
2
points. When the distance between two points
3 km v
decreases, they are said to approach each other. The
rate at which the points ‘kill’ their separation is called
velocity of approach. To understand the concept of
60 velocity of approach and to devise a method for its
3 km 1 km
computation, we consider a few motions of two points
described in the examples below.
1 (i) Two points A and B move along a straight line in
dmin opposite directions with speeds 3 m/s and 2 m/s, as
shown in Fig. 4.79(a).
Fig. 4.78

97
5 m/s 10 m/s
A 5 m/s
3 m/s 2 m/s A 2 unit
5 m/s
5 m/s
A B B
(a)
3 m/s 2 m/s B 5 m/s 1 units
10 m/s
5 m/s B
A B
(b) (c) 5 m/s
5 m/s
(a) (b)
Fig. 4.79
In the given motion, points A and B come closer by 5 m Fig. 4.81
in every one second time until they cross each other; the
angular velocities about the centre is same. If you plant
separation between them decreases by 5 m every
yourself at B and look at point A, what will you
second. We can express this by saying that the velocity
observe? Point A describes a circular path? An elliptical
of approach between points A and B is 5 m/s.
path? Or some other complicated path? Note that the
(ii) In Fig. 4.79(b), points A and B have velocities 3 m/s distance between points A and B does not change.
and 2 m/s in the same direction along a straight line. The (vi) In Fig. 4.81(b), the points A and B move with speed
distance between points A and B will decrease by 1 m 10 m/s and 5 m/s, respectively, their velocity vectors
every second as long as point A is behind B. Velocity of always being in opposite directions and also always
approach between points A and B is 1 m/s. If you draw perpendicular to the line AB. What about the separation
their positions at instants 1 s, 2 s, 3 s…, you find that the between the two points? Will it change or not? What is
separation between them decreases by 1 m every second. the velocity of approach between the points?
(iii) In Fig. 4.79(c), points A and B move with equal and (vii) Point A moves such that it has two components of
constant velocities, their velocities always being velocity: one component is 5 m/s, always perpendicular to
perpendicular to the line connecting them. Clearly, the the line AB, and the other component is 2 m/s along AB.
separation between the points remains constant. The
t=0
velocity of approach between them is zero, va = 0. A 5 m/s
(iv) Consider the velocities of points A and B as shown 2 m/s t=t
in Fig. 4.80. Velocity of approach of points A and B is 5 m/s 2 m/s
2 m/s + 3 m/s = 5 m/s. How?
It can be shown that points A and B come closer by 5 m 5 m/s
every second. For how long will they do it? When will 1 m/s
they start moving apart? 1 m/s
You must not miss the most crucial information in this 5 m/s B
example. The velocity components of the points that are
perpendicular to the line joining them are equal and in Fig. 4.82
the same direction. Point B also has two components of velocity: one
A 5 m/s component is 5m/s always perpendicular to AB as
2 m/s shown in Fig. 4.82 and other components is 1 m/s along
A AB aimed at A. Velocity of approach between points A
5 m/s
2 m/s and B is 3 m/s. How?
3 m/s (viii) In Fig. 4.83, the points A and B are 15 m apart
5 m/s initially.
3 m/s B
6m
B 5 m/s A 6 m/s
9m
6 m/s
Fig. 4.80
9 m/s
(v) Points A and B move with a speed 5 m/s each, such that 45
15 m 5m
their velocity vectors are in opposite directions and always 1 m/s
9 m/s
perpendicular to the line AB as shown in Fig. 4.81(a). 1 m/s
45
The length of the line segment AB will not decrease in 1 m/s 1m
this case. Velocity of approach between the points is B 1 m/s
zero. Both the points move in a circle, the mid point
1m 5m
between them being the centre. It can be seen that their
(Not on Scale)
Fig. 4.83

98
Point A moves with constant velocity, with components body approaches the Sun with velocity v . One should
6 m/s and 9 m/s as shown. Point B also moves with a not have any doubt that the velocity of approach
constant velocity with components 1 m/s and 1 m/s. At between the cosmic body and the Sun is a variable
the initial moment (t = 0), the velocity of approach is 9 quantity in example shown in the figure. Can you
m/s + 1 m/s = 10 m/s. predict the velocity of approach when the cosmic body
What is the velocity of approach between points A and is closest to the Sun? When the cosmic body is closest
B at the moment of time t = 1 s? to the Sun, what is the angle between its radius vector
To calculate the velocity of approach at t = 1 s, draw from the Sun and its velocity vector?
the positions of points A and B at this moment. And
then resolve the velocities of A and B along the line AB (xi) Depicted in Fig. 4.86 is the motion of two points
and perpendicular to it. Displacements, distances and A and B along two parallel lines. The points come to
angles that matter are shown in Fig. 4.84. the closest at moment t0.

6 cos 45 A v1 A A v1 cos 2


2
1 v1
6 m/s
9 cos 45 v1 cos 1
3 cos 45 v2 cos 1
9 m/s
1 m/s
1 cos 45 v2 1 B
1 cos 45 2 cos 45 2 B B v2
1 m/s t < t0
v2 cos 2 t > t0 t0
Fig. 4.84
Fig. 4.86
The velocity of approach between points A and B at
t = 1 s is va = 3 cos 45 + 2 cos 45 = 5 cos 45 m/s. At For time t  t0 , the points come closer, velocity of
what moment of time is the velocity of approach a approach between them being va  v1 cos 1  v2 cos 1.
between A and B maximum? When do points start How?
getting separated?
(ix) 4.7.1. Velocity of Separation
When the distance between two points, that is, their
A
separation increases, we speak of velocity of separation,
3 sin 30 Sun which is defined as the rate at which the separation
3 cos 30
3 m/s 30 increases.
v||
The examples that follow illustrate the concept of
60
2 cos 60 velocity of separation vividly.
B
2 m/s v0 (i) In the motion depicted in Fig. 4.87(a), the velocity of
2 sin 60 v separation between points A and B is 5 m/s.
(a) (b) (ii) The velocity of separation between points A and B
in the motion shown in Fig. 4.87(b) is 1 m/s.
Fig. 4.85 (iii) Points A and B move with constant velocities as
Points A and B move in perpendicular directions as in shown in the Figure 4.87(c). The velocity of separation
Fig. 4.85(a). At a certain moment of time their positions is 3 m/s + 2 m/s = 5 m/s. You must note that the
and velocities are shown in the figure. velocity components perpendicular to the line joining
Velocity of approach between the points A and B at this points A and B are equal.
moment is (3 cos 30 + 2 cos 60) m/s. 3 m/s

(x) A cosmic body is approaching the Sun along a path A B 2 m/s A 5 m/s
that can be approximated to be elliptical, Fig. 4.85(b). 3 m/s
On looking at the cosmic body from the Sun, one finds (a)
that the length of line of sight to the body shrinks, the A 2 m/s B
body comes nearer to the Sun. In the figure, the velocity B 5 m/s
3 m/s
of the cosmic body has been resolved into two (b)
2 m/s
components - the component v along the line of sight, (c)

and the component v perpendicular to it. The cosmic


Fig. 4.87

99
(iv) Points A and B move with constant velocities as separation of the points? By a look at the velocity
shown in Fig. 4.88. components along the line joining the points, we arrive
1 m/s at the answer, vs  v1 cos 2  v2 cos 2 .
1 m/s A 45 5 m/s (vii) Let us consider Example (x) of the previous
A
section, Fig. 4.85(b). After kissing the Sun the cosmic
5 m/s 45 body moves away from it. The separation between the
1m 4m cosmic body and the Sun now increases.
1 m/s v||
B B 45
1 m/s v0
2 m/s
2 m/s
t=0 v
4m
Fig. 4.88
Sun
At the initial moment t = 0, the points A and B are 1 m
apart. What is the initial velocity of separation? It is Fig. 4.91
3 m/s. What is the velocity of separation at t = 1 s? It follows from Fig. 4.91 that the velocity of separation is
Mark the positions of A and B at t = 1 s. Calculate the v , the component of velocity on the line connecting the
relevant distances and angles, as depicted in Fig. 4.89.
cosmic body to the Sun. What will be the velocity of
Find the components of velocities along the line AB.
separation when the cosmic body is farthest from the Sun?
5 cos 45 m/s
 Example 25. Let us take up Example 30, Chapter 3
1 cos 45
A again. We shall apply the concept of velocity of
5 m/s approach to this problem.
6 cos 45 m/s The particles 1 and 2 come closer for some time and
1 cos 45 m/s A then they go farther apart. Figure 4.92 shows the
45
 configuration at time t . The distance between them
B
1 m/s will be the least at the moment the approach velocity is
2 cos 45 45 B zero. That is, v1 cos   v2 sin   0
2 m/s
1 cos 45
Fig. 4.89 v2t
The velocity of separation between the points A and B 2 l2
at time t = 1 s is v2 sin 
vs = 1 cos 45 + 6 cos 45 = 7 cos 45 m/s. v1 v2 t=0
1 
(v) Points A and B move such that one component of v1 cos 
velocity of A is perpendicular to the line AB and the l1
other along AB directed away from B, and one v1t
component of velocity of B is perpendicular to AB and
the other is along AB directed away from A, Fig. 4.90. Fig. 4.92
4 m/s v1
4 m/s
A or tan   . …(i)
v2
5 m/s
A Also from the figure,
5 m/s l2  v2t
tan   . …(ii)
v1t  l1
5 m/s

5 m/s
B From Eqs. (i) and (ii), we get
l2  v2t v1
2 m/s
B 2 m/s 
v1t  l1 v2
Fig. 4.90 v1l1  v2l2
or t .
Velocity of separation between points A and B is v12  v2 2
4 m/s + 2 m/s = 6 m/s. Hence the least distance between the particles,
(vi) For the motion sketched in Fig. 4.86, the points
move apart for time t  t0 . What is the velocity of

100
2 2 A
 v l v l   l1v1  l2 v2 
lmin   l2  v2  1 12 2 22    v1  2  l1 
 v1  v2 v  v 2 
   1 2  A
| v1l2  v2 l1 |
 . v sin60
v12  v2 2 v cos 60
B C
v v sin60
Example 26. Three points are located at the vertices of B
an equilateral triangle whose side equals a. They all v v cos60 C
start moving simultaneously with constant speed v, with
Fig. 4.94
the first point heading continually for the second, the
second for the third and the third for the first. How soon Example 27. Point A moves uniformly with speed v so
will the points converge? 
that the vector v is continually ‘aimed’ at point B
To understand the motion of points, we treat the which in its turn moves rectilinearly and uniformly with
velocity of each point as constant for time interval dt  
speed u < v. At the initial moment of time v  u and
and calculate the infinitesimal displacement. We will do
it for the next dt, and then again for the next dt. When points are separated by a distance l. How soon will the
we draw the positions of the points after dt’s we get the point converge?
pattern of motion illustrated in Fig. 4.93. Figure 4.95 sketches the motion of points A and B. If
A the points converge at point C, say at time t = T, the x-
component of displacement of A must be equal to the
displacement of B and the y- component of the
displacement of A must be equal to l. The x and y
components of velocity of A are vx = v cos  and
vy = v sin . You can find the x- and y- components of
displacement of A from the following equations.
O y
u u C
B
v1 v1 t=T
v1 v2
v1 l  vx = v cos 
B C A vx vy = v sin 
vdt B1 B2 v2 v
v2
v2 v2
A x
t=0
Fig. 4.93
Fig. 4.95
Now let us focus on the motion of point B. If v1 and v2
T T
are components of velocity of B along BO and
perpendicular to it at any instant, v1 = v cos 30 and v2 
x  vx dt
0
and y   v y dt.
0
= v sin 30. This can be inferred from the symmetry of
the problem and simple geometrical calculations. Substitution of vx = v cos  and vy = v sin  into the
Hence, we can assert that point B approaches point O at integrals, and the condition of convergence of points A
a constant rate of v sin 30. This gives and B give the following two equations
T
Initial separation
Time of convergence 
Velocity of convergence
 v cos  dt  uT … (i)
0
 (a / 2)  T


 cos 30  2a
 .
and  v sin  dt  l. …(ii)
0
v cos 30 3v
Now what is left to be done is to solve these two
We can also arrive at the answer by analyzing the
equations for T.
convergence of points B and C, see Fig. 4.94. It can be
shown that B and C converge at a constant rate of Acknowledging that  is a variable quantity, one can
anticipate that solving for T from these equations is a
v  v cos 60 until they meet. Hence, the time of formidable task. A different and easier approach to the
a 2a solution is sought.
convergence t   .
v  (v / 2) 3v The concept of velocity of approach can be tried.
Velocity of approach (or velocity of convergence)

101
between points A and B is (v – u cos ) which is also a 1m
B
variable quantity. See Fig. 4.96.
u cos 
B u C A
v

l  1m
A

Fig. 4.97
Fig. 4.96 The balls interchange their velocities, that is, ball B stops and ball A
takes off with the velocity with which ball B hits it. For the time
You can calculate the distance of convergence (the being, let us use this information and proceed. You will learn about
separation they ‘kill’) in an infinitesimally small time dt collisions in Chapter 9 in details.
by treating v – u cos  as constant over this interval. (i) Comparison of times of motion of balls:
This infinitesimal distance of convergence equals The vertical acceleration of ball A falling from the table
(v – u cos )dt. Integration of this infinitesimal distance is always g, therefore, the time it takes to fall 1 m can
over total time of motion gives the total distance by be calculated as
which the points A and B converge, which is, obviously,
1
equal to the initial separation l. That is, 1m   9.8 m/s2  t 2
T
2
l   (v  u cos ) dt. …(iii) which gives t  0.5 s (approximately).
0
How long does ball B take in moving though the circular
arc? The calculation of the time of motion of ball B
Eqs. (i) and (iii) can be easily solved for the unknown requires an integration to be evaluated.
T. Multiply Eq. (i) by u and Eq. (iii) by v and then add But it turns out that you can answer this question without
the resulting equations. This gives actually calculating the time of motion of ball B. What
T
can be stated with certainty is that, since the thread exerts
u 2T  vl   v 2 dt an upward force on ball B, its vertical acceleration is
0
always less than g. Therefore, the vertical motion of ball
or u T + vl = v2T
2
B takes a longer time than the vertical free fall of ball A;
vl ball B stays in motion for longer.
or T 2 .
v  u2 (ii) Comparison of distances covered by the balls:
If point A moves with the same speed as that of point The bob of the pendulum - ball B - describes one fourth
B, the points moving in the same fashion as described of a circle. The distance it covers is
in this problem, A will not be able to catch B. r 3.14  1m
  1.5 m. The other ball, A, follows a
Eventually both A and B will be moving on the given 2 2
straight line, one point behind the other. What is the parabolic path, the length of which cannot be
final separation between the points? determined by elementary methods, it requires an
Another very interesting problem on the concept of integration. However, you can find out which ball
velocity of approach is discussed in Example 37.  covers a longer distance without actually calculating the
distance covered by ball A. This ball hits the ground at a
Additional Examples 2 1 m
distance of vt  2  g  1m   2 m from the
g
Example 28. A small steel ball A is at rest on the edge
of a table of height 1 m. Another steel ball B, used as edge of the table. The length of its path is, therefore, not
the bob of a metre-long simple pendulum, is released less than the distance between the starting point and the
from rest with the pendulum suspended horizontal, and point where it hits the ground, which is 5 m  2.2 m.
swings against A as shown in Fig. 4.97. The masses of In summary, ball A moves on a longer path, but stays in
the balls are identical and the collision is elastic. motion for a shorter time than ball B.
Considering the motion of A only up until the moment
Example 29. A particle of mass m carries an electric
it first hits the ground,
charge q and is subject to the combined action of
(i) which ball is in motion for the longer time;
gravity and a uniform horizontal electric field of
(ii) which ball covers the greater distance?
strength E. It is projected with speed v in the vertical
plane parallel to the field and at an angle  to the

102
horizontal. What is the maximum distance the particle t in Eq. (i), we obtain
can travel horizontally before it is next level with its 2v sin  1  4v 2 sin 2  
starting point? R  (v cos )  aH  
You have investigated the motion of a projectile on an g 2  g2 
inclined plane. You have also calculated the maximum v2 a v2
range of the projectile along the plane. This problem is or R sin 2  H 2 (1  cos 2) .
g g
very similar to inclined plane projectile problem. Figure
4.98 depicts the similarity and also the differences in dR
Now,  0 , gives
motion in the two cases. d
Since the particle carries a change q, in electric field of E
2v 2 2 a v2
(horizontal); there is a horizontal acceleration of aH 
qE
in the cos 2  H2 sin 2  0
m g g
particle. (This is additional information which you haven’t learnt yet.
g
The concept of electric field will be introduced in Volume 3 of this
text. The expression for horizontal acceleration should not frighten
or tan 2   . …(iii)
aH
you.) The rest is just about doing simple kinematics under constant
horizontal acceleration aH along with vertical acceleration g due to For the value of  given by Eq. (iii), the distance particle
gravity. travels horizontally before it is next in level with its
starting point is maximum. An interesting point must be
g sin g cos  y qE noted here. The value of tan 2 is negative; this means
v aH 
m that 2 lies in the second quadrant, which implies
y v v sin  g 
x   .
x 4
 v cos  g 2  aH2

Now we can calculate g
the maximum value of 2
v0 x  v cos , a x   g sin  qE the horizontal distance as –aH
v0 x  v cos , ax 
v0 y  v sin , a y   g cos  m
v0 y  v sin , a y   g   
v2 g a v2  aH
Rmax    H 2 1   
Fig. 4.98 g g  aH   
  g  aH
2 2 g 2 2

As we do in all projectile motions, we will analyze the
two components of the motion - along x- axis and along After some simple algebra, the above equation
simplifies to
y- axis separately and independently. Let the particle hit
the ground at point P such that OP  R, Fig. 4.99.
qE
v2

Rmax  2 aH  g 2  aH2 .
g

aH 
y m
v qE
g On substituting aH  into the expression for Rmax
v sin 
m
 we get
O x
v cos  P  qE 2 
v2   qE  
R Rmax   g2   
g2  m  m  
Fig. 4.99  
For the x- component of motion:
1
R  (v cos )t  aH t 2 . …(i)

v2
mg 2
 qE  m g 2  q2 E 2 .
2
At point P the y- component of displacement is zero. Example 30. Let us make the problem of Example 24 a
Therefore, for the y- component of motion: bit more interesting. Assume the position of the balloon
at t = 3 s as the origin of a coordinate system with x-
1
0  v sin t  gt 2 . …(ii) axis in the horizontal and y- axis in the vertical
2 direction. A wall has been constructed whose
From these equations we will calculate R in terms of intersection with the trajectory of ball (II) is described
 ; and then set
dR
 0 to determine  for which R is with by equation y = h  x tan , where  = 30, and
d 1
maximum. h  m as shown in Fig. 4.100. At what point (x, y)
2
2v sin  does the ball strike the wall?
Eq. (ii) gives t  . On substituting this value of
g

103
Equation of the above mentioned line on the wall is Let the ball be projected at an angle  with the
y = h  x tan  horizontal and strike the wall in time t.
1 x The laws of motion for the ball are
or y  . x = (v cos )t
2 3
1
y y = (v sin)t  gt 2 .
2
Wall Equation of the wall is
y = h  x tan .
y = h – x tan 
On eliminating x and y from these equations we
3 m/s  = 30o obtain
x 1 2
(0, 0) 1 m/s gt  v [cos  tan   sin ] t  h  0
2
Fig. 4.100 1 2  cos  sin   sin  cos  
or gt  v  th0
The equations of motion of the ball (II) are 2  cos  
x  (1m/s)t  t 1 2 v
or gt  [sin(   )] t  h  0. …(i)
1 2 cos 
y  (3m/s) t  (10 m/s 2 )t 2  3t  5t 2 .
2 Differentiating the above equation relative to ,
Substituting for t from the first equation into the
second, we obtain equation of the trajectory 1 dt v  dt 
g  2t  t cos(   )  sin(  )   0
y = 3x – 5x2. 2 d  cos   d
On solving the equation of the line with the equation of dt dt
trajectory, we obtain For t to be minimum,  0. On substituting 0
d d
1 x
  3x  5x2 in the above equation we obtain
2 3 v
1  [cos(  )] t  0
or (5 3) x 2  (1  3 3) x  3  0. cos 
2 or cos(  )  0
Roots of the above quadratic are
  +  = 90
x  0.23 m and 0.56 m.
or  = 90  .
Two values of x arise because the trajectory of the ball Substituting  = 90 –  into Eq. (i) and solving for t
(II) is parabolic which would have intersected we obtain
the line y = h  x tan  at two points had this been just
a line, not a line on the wall. In the given arrangement v  v 2  2 gh cos 2 
tmin  .
ball will hit the wall only once, so only the first impact g cos 
point is to be considered for which x = 0.23 m, and It can be seen that tmin will have a real, meaningful
correspondingly
value only if
1 0.23
y   0.37 m. v 2  2 gh cos2 
2 3
Ball strikes the wall at point (0.23, 0.37) m. or v  2 gh cos  .
Example 31. At what angle with horizontal should a Example 32. On a frictionless horizontal surface,
particle be thrown from the origin O(0,0) with a assumed to be the x-y plane, a particle A is moving
velocity v relative to the ground so that it takes along a straight line parallel to y- axis with a constant
minimum time to hit the line y  h  x tan , see velocity of ( 3  1) m/s. At a particular instant when
Fig. 4.101? Also, find the limiting value of v. the line OA makes an angle of 45 with the x- axis,
y another particle B is projected along the surface from
(0, h)
the origin O, see Fig. 4.102. Its velocity makes an angle
 with the x- axis and it hits particle A. (a) How must
v y = h – x tan  the velocity v of particle B be related to the angle 
for this to happen? Can you determine the values of 
  x
O and v uniquely on the basis of forgoing given
Fig. 4.101 information? (Continued….)

104

You might also like